Vous êtes sur la page 1sur 105

Musculoskeletal Trauma Scored and Recorded Self-Assessment Examination 2015

Musculoskeletal Trauma Scored and Recorded Self-Assessment Examination 2015

Musculoskeletal Trauma Scored and Recorded Self-Assessment Examination 2015

Figure 1a

Figure 1b

CLINICAL SITUATION FOR QUESTIONS 1 THROUGH 3


Figures 1a and 1b are the radiographs of a 70-year-old retired man who falls
while skiing and injures his right hip. He had no preceding hip pain. After
the fall, he is unable to ambulate and is transferred down the mountain by
the ski patrol and taken to a hospital.
Question 1 of 101
The major blood supply to the femoral head comes from which vessel?
1234-

Lateral femoral circumflex artery


Medial femoral circumflex artery
Artery of the ligamentum teres
Inferior gluteal artery

PREFERRED RESPONSE: 2 -

Medial femoral circumflex artery

Musculoskeletal Trauma Scored and Recorded Self-Assessment Examination 2015

Question 2 of 101
A formal multidisciplinary team approach to the comanagement of geriatric
patients with hip fracture has been shown to lead to
1234-

decreased intraoperative blood loss.


decreased surgical time.
decreased inpatient mortality.
decreased per-patient costs.

PREFERRED RESPONSE: 4 -

decreased per-patient costs.

Question 3 of 101
Which factor is a potential disadvantage of total hip arthroplasty compared
to hemiarthroplasty for treatment of displaced femoral neck fracture in older
patients with higher functional demands?
1 - Increased long-term overall costs
2 - Increased risk for dislocation
3 - Increased risk for revision surgery
4 - Decreased postsurgical function
PREFERRED RESPONSE: 2 - Increased risk for dislocation
DISCUSSION
The main source of blood supply to the femoral head is the deep branch of
the medial femoral circumflex artery. The lateral femoral circumflex artery
and artery of the ligamentum teres contribute to a lesser degree, while the
inferior gluteal artery has a minimal contribution. This vascular supply is
compromised with displaced femoral neck fractures and results in a high rate
of osteonecrosis. This is a reason to consider arthroplasty for older patients
who may not be able to tolerate multiple procedures.
Studies evaluating comanagement protocols for the treatment of hip
fractures in patients older than age 60 have demonstrated significant
improvements in mortality, length of stay, complication and readmission
rates, and ambulatory status at time of discharge while decreasing costs.
3

Musculoskeletal Trauma Scored and Recorded Self-Assessment Examination 2015

Surgical time, blood loss, time to surgery, and inpatient mortality have not
been altered.
Total hip arthroplasty is more frequently recommended for primary
treatment of displaced femoral neck fractures in older, active patients who
would have otherwise been treated with hemiarthroplasty. Risk for
acetabular erosion is alleviated, implant survival is longer, and revision
surgery rates are lower, as are overall long-term costs. Postsurgical function
is not compromised and may actually be better. Dislocation rates are
increased (up to 10%), although these rates may be lessened with recent
improvements in component design that allow for use of larger femoral
heads.

Musculoskeletal Trauma Scored and Recorded Self-Assessment Examination 2015

Musculoskeletal Trauma Scored and Recorded Self-Assessment


Examination 2015

Figure 4a

Figure 4b

Figure 4c

Question 4 of 101
A 30-year-old man was involved in a high-speed motorcycle collision and
sustained the injury shown in Figure 4a. Hypotension ensued shortly after
arrival in the emergency department. Figure 4b is the initial contrast pelvic
CT image with an unrecognized blush consistent with arterial bleeding.
During surgical repair, the patient was noted to have active bleeding and an
angiogram was obtained (Figure 4c). Which structure is the likely cause of
his bleeding?
1 - Superior gluteal artery
2 - Branch of the external iliac artery
3 - Branch of the pudendal artery
4 - Branch of the femoral artery
PREFERRED RESPONSE: 3 - Branch of the pudendal artery
DISCUSSION
Pelvic bleeding occurs predominantly from disruption of the posterior
venous plexus and bleeding from the fractured bone. Occasionally arterial
bleeding is seen, with injury to the superior gluteal artery most common.
Anterior pelvic bleeding occurs from injury to the obturator artery
(commonly from a pubic bone fracture laceration) and less frequently from
the pudendal artery near the symphysis. The location of the bleeding on CT
and angiography images does not correspond to the superior gluteal, external
iliac, or femoral arteries.
5

Musculoskeletal Trauma Scored and Recorded Self-Assessment Examination 2015

Musculoskeletal Trauma Scored and Recorded Self-Assessment Examination 2015

Figure 5a

Figure 5b

RESPONSES FOR QUESTIONS 5 THROUGH 8


1 - Avascular necrosis, head collapse, and screw penetration
2 - Fixation failure and varus collapse
3 - Humeral stem loosening
4 - Glenoid component loosening
5 - Hardware failure (breakage of plate or screws)
6 - Shoulder dislocation
Please choose from the responses to identify the most likely complication in
each scenario.

Musculoskeletal Trauma Scored and Recorded Self-Assessment Examination 2015

Question 5 of 101
An active 79-year-old woman with the radiograph and intraoperative image
shown in Figures 5a and 5b undergoes open reduction and internal fixation
(ORIF) of her proximal humerus fracture.
1 - Avascular necrosis, head collapse, and screw penetration
2 - Fixation failure and varus collapse
3 - Humeral stem loosening
4 - Glenoid component loosening
5 - Hardware failure (breakage of plate or screws)
6 - Shoulder dislocation
PREFERRED RESPONSE: 1 - Avascular necrosis, head collapse, and
screw penetration
Question 6 of 101
A 73-year-old woman sustains a displaced 3-part proximal humerus fracture.
At the time of surgery, she has a massive rotator cuff tear in addition to the
proximal humerus fracture. She is treated with total shoulder arthroplasty
(TSA).
1 - Avascular necrosis, head collapse, and screw penetration
2 - Fixation failure and varus collapse
3 - Humeral stem loosening
4 - Glenoid component loosening
5 - Hardware failure (breakage of plate or screws)
6 - Shoulder dislocation
PREFERRED RESPONSE: 4 - Glenoid component loosening

Musculoskeletal Trauma Scored and Recorded Self-Assessment Examination 2015

Question 7 of 101
An 82-year-old woman with osteoporosis has increased pain and difficulty
using her arm 3 weeks after undergoing ORIF of her 4-part proximal
humerus fracture.
1 - Avascular necrosis, head collapse, and screw penetration
2 - Fixation failure and varus collapse
3 - Humeral stem loosening
4 - Glenoid component loosening
5 - Hardware failure (breakage of plate or screws)
6 - Shoulder dislocation
PREFERRED RESPONSE: 2 - Fixation failure and varus collapse

Question 8 of 101
A 79-year-old woman with a massive rotator cuff tear presents to the
emergency department with pain and difficulty moving her arm 7 weeks
after undergoing reverse TSA for a displaced 4-part proximal humerus
fracture.
1 - Avascular necrosis, head collapse, and screw penetration
2 - Fixation failure and varus collapse
3 - Humeral stem loosening
4 - Glenoid component loosening
5 - Hardware failure (breakage of plate or screws)
6 - Shoulder dislocation
PREFERRED RESPONSE: 6 - Shoulder dislocation

DISCUSSION
The complication rate is high after surgical treatment of proximal humerus
fractures, particularly in elderly patients with osteoporotic bone. In patients
treated with ORIF, common complications include varus malunion (16%),
avascular necrosis (10%), screw penetration (8%), and infection (4%). In
8

Musculoskeletal Trauma Scored and Recorded Self-Assessment Examination 2015

cases involving a dislocation of the humeral head, avascular necrosis is more


common. In patients treated with hemiarthroplasty or TSA, complications
include component loosening, infection, and dislocation. TSA is associated
with glenoid loosening in patients with rotator cuff incompetence and should
be avoided in these patients. Reverse TSA is a potential solution for this
population. Dislocation and postoperative infection are potential
complications after reverse TSA.

Musculoskeletal Trauma Scored and Recorded Self-Assessment Examination 2015

Musculoskeletal Trauma Scored and Recorded Self-Assessment


Examination 2015

Figure 9a

Figure 9b

Figure 9c

Figure 9d

Question 9 of 101
Figures 9a through 9d are the radiographs of a 21-year-old woman who is
involved in a high-speed motor vehicle collision and sustains an isolated
right closed-foot injury. Before surgery, the patient is advised about the
relatively poor long-term outcomes associated with this injury. What is the
most common reason for functional limitations after surgical treatment in
this scenario?
10

Musculoskeletal Trauma Scored and Recorded Self-Assessment Examination 2015

1 - Subtalar arthritis
2 - Osteonecrosis
3 - Nonunion
4 - Varus malunion
PREFERRED RESPONSE: 1 - Subtalar arthritis
DISCUSSION
When a displaced talar neck fracture occurs, the rate of osteonecrosis is
high; however, many revascularize the talus without collapse. A nonunion
can occur but is less common than osteonecrosis and arthritis. A varus
malunion can be debilitating and lead to subtalar arthritis. In a fracture with
the talar body dislocated posteromedially (such as in this example)
neurologic deficits in the tibial nerve distribution are common but typically
improve with urgent reduction. Studies show that posttraumatic subtalar
arthritis is common after this injury and is the most likely cause of long-term
functional impairment.

11

Musculoskeletal Trauma Scored and Recorded Self-Assessment Examination 2015

Musculoskeletal Trauma Scored and Recorded Self-Assessment


Examination 2015

Figure 10a

Figure 10b

Figure 10c

CLINICAL SITUATION FOR QUESTIONS 10 THROUGH 12


Figure 10a is the radiograph of a 30-year-old man who sustained an injury in
a motor vehicle collision.
Question 10 of 101
This patient underwent fixation and his radiographs (Figures 10b and 10c) at
6 weeks are shown. What was the failure mode for this implant?
1 - Varus collapse and hardware failure
2 - Screw cut out in the femoral head
3 - Failure of distal screws and loss of fixation
4 - Lack of patient compliance
PREFERRED RESPONSE: 1 - Varus collapse and hardware failure

12

Musculoskeletal Trauma Scored and Recorded Self-Assessment Examination 2015

Question 11 of 101
The biomechanical reason for implant failure in this case is related to
1 - lack of medial cortical contact secondary to comminution.
2 - lack of friction fit of plate to bone.
3 - varus malreduction of the fracture.
4 - poor bone quality in the femoral head and diaphysis.
PREFERRED RESPONSE: 1 - lack of medial cortical contact secondary
to comminution.

Question 12 of 101
Among the options listed below, what is the best treatment for the
complication shown in Figure 10c?
1234-

Removal of hardware and bone grafting


Removal of hardware and total hip arthroplasty (THA)
Removal of hardware and revision using a first-generation femoral nail
Removal of hardware and revision using a second-generation femoral nail

PREFERRED RESPONSE: 4 a second-generation femoral nail

Removal of hardware and revision using

DISCUSSION
Proximal femur fractures can be treated using a variety of implants including
intramedullary nails, blade plates, and locking plates (now precontoured
proximal femur plates). The comminution and lack of medial cortical
support may predispose these fractures to nonunion.
The recent popularity of locking plates for proximal femur treatment has
increased their use for this fracture; however, a disproportionately high rate
of failure of these plates, including early implant failure with plate and screw
breakage, cut out with varus collapse, and nonunion have been reported.?

13

Musculoskeletal Trauma Scored and Recorded Self-Assessment Examination 2015

Malreduction predisposes these fractures to failure. The initial postoperative


radiographs do not reveal a varus malreduction because the tip of the greater
trochanter is below the center of the femoral head.
Once failure occurs, the best fixation method among the options detailed is
an intramedullary nail (second generation with screws into the femoral head)
and restoration of alignment. THA is usually not recommended for treatment
of subtrochanteric femur fractures in young patients.

14

Musculoskeletal Trauma Scored and Recorded Self-Assessment Examination 2015

Musculoskeletal Trauma Scored and Recorded Self-Assessment


Examination 2015
Question 13 of 101
Which medication or supplement is recommended to promote healing of
atypical subtrochanteric fractures?
1234-

Bisphosphonates
Teriparatide
Vitamin D
Glucosamine chondroitin

PREFERRED RESPONSE: 2 -

Teriparatide

DISCUSSION
Use of teriparatide in association with fracture fixation promotes healing
because these fractures are associated with delayed healing. The other
responses are not associated with healing of these fractures.

15

Musculoskeletal Trauma Scored and Recorded Self-Assessment Examination 2015

Musculoskeletal Trauma Scored and Recorded Self-Assessment


Examination 2015

Figure 14a

Figure 14b

estion 14 of 101
An 18-year-old man was involved in an altercation during which he
sustained the injuries shown in Figures 14a and 14b. His Glasgow Coma
Scale (GCS) score is 11 (a GCS score of 9-12 indicates moderate head
injury). The neurosurgeons elect to not place an intracranial pressure (ICP)
monitor. The patient responds appropriately to stimuli and is
hemodynamically stable. What is the most appropriate initial treatment?
1 - Knee immobilizer
2 - Immediate spanning external fixation
3 - Immediate intramedullary nailing
4 - Immediate plate fixation
PREFERRED RESPONSE: 2 - Immediate spanning external fixation

16

Musculoskeletal Trauma Scored and Recorded Self-Assessment Examination 2015

DISCUSSION
Although management of femoral shaft fractures in patients with head
injuries remains controversial, most practitioners agree that "damage-control
principles" are appropriate for patients with evolving head injuries. This
patient has a subarachnoid hemorrhage and a decreased GCS but is
responding appropriately. The best treatment is a damage-control approach
for the femur that will cause minimal blood loss and allow the brain injury
(and swelling) to equilibrate. External fixation can be performed
expeditiously and with minimal blood loss, which will reduce further injury
to the brain. Special attention should be paid to maintaining cerebral
perfusion pressure higher than 70 mmHg. Admission to the intensive care
unit is recommended for monitoring of this injury. Knee immobilizers are
not tolerated well by young muscular men with femur shaft fractures. A
GCS score of 11 or higher can be observed without ICP monitoring.

17

Musculoskeletal Trauma Scored and Recorded Self-Assessment Examination 2015

Musculoskeletal Trauma Scored and Recorded Self-Assessment


Examination 2015

Figure 15
Question 15 of 101
The most common reason for proximal femur fracture fixation failure
(Figure 15) is secondary to which common deformity?
1234-

Varus
Valgus
Malrotation
Shortening

PREFERRED RESPONSE: 1 -

Varus

DISCUSSION
Malposition of a proximal lag screw may result in cut-out similar to that
seen with a sliding hip screw. Varus malreduction also can result in implant
failure. Studies have shown no difference in complication or healing rates
when comparing short and long cephallomedullary nails.
18

Musculoskeletal Trauma Scored and Recorded Self-Assessment Examination 2015

Musculoskeletal Trauma Scored and Recorded Self-Assessment


Examination 2015
CLINICAL SITUATION FOR QUESTIONS 16 THROUGH 20
A 23-year-old man sustains multiple injuries in a high-speed motor vehicle
collision. Among his injuries are a right transverse-posterior wall acetabular
fracture, a left open tibia fracture with compartment syndrome, and a right
calcaneus fracture.
Question 16 of 101
After initial evaluation he is taken to the operating room urgently and
undergoes debridement of his open tibia fracture, 4-compartment
fasciotomy, and intramedullary nailing of the fracture. Negative pressure
wound therapy (NPWT) is chosen for the open wound and fasciotomy sites.
NPWT in this scenario will
1 - remove bacteria from the wound and decrease risk for infection.
2 - promote wound contraction, making primary closure less likely.
3 - promote local wound perfusion.
4 - decrease compartment pressures.
PREFERRED RESPONSE: 3 - promote local wound perfusion.

19

Musculoskeletal Trauma Scored and Recorded Self-Assessment Examination 2015

Question 17 of 101
Nine hours after surgery you are contacted because the patient has continued
tachycardia and ongoing resuscitation needs. The NPWT canister has been
emptied 3 times in the last 8 hours and contains sanguinous fluid. In addition
to continued resuscitation, what is the most appropriate next step??
1 - Order the NPWT applied to wall suction to allow less frequent emptying
of the canister.
2 - Clamp off the suction device and return to the operating room for wound
exploration.
3 - Turn the suction down from -125 mm Hg to -50 mm Hg.
4 - Take the patient for angiography and possible embolization.
PREFERRED RESPONSE: 2 - Clamp off the suction device and return to
the operating room for wound exploration.
Question 18 of 101
On postinjury day 3 the patient undergoes open reduction and internal
fixation of his right acetabular fracture via a Kocher-Langenbeck approach.
On postoperative day 5 he is noted to have persistent serous drainage
without any localized signs of infection. Incisional NPWT used in this
setting would likely result in
1. infection.
2. a sealed wound (more rapidly than sealing would occur with a
compressive dressing).
3. hematoma formation.
4. it can electively delay flap coverage for 3 to 4 weeks.
PREFERRED RESPONSE: 2 - a sealed wound (more rapidly than sealing
would occur with a compressive dressing).

20

Musculoskeletal Trauma Scored and Recorded Self-Assessment Examination 2015

Question 19 of 101
The patient undergoes repeat debridements for the open tibia fracture and
associated compartment syndrome. It becomes apparent that the medial open
fracture wound is not amenable to primary closure. NPWT is useful in this
setting because
1 - it will stabilize the soft-tissue environment while the patient awaits
flap coverage.
2 - it will promote granulation of the wound over the exposed fracture site
to prevent flap coverage.
3 - it will promote fracture healing.
4 - it can electively delay flap coverage for 3 to 4 weeks.
PREFERRED RESPONSE: 1 - it will stabilize the soft-tissue environment
while the patient awaits flap coverage.

21

Musculoskeletal Trauma Scored and Recorded Self-Assessment Examination 2015

Question 20 of 101
The patient subsequently requires split-thickness skin grafting over his
lateral fasciotomy wound during soft-tissue reconstruction. In this setting,
NPWT
1 - will likely improve incorporation of the graft.
2 - will provide an inconsistent bolster to the graft.
3 - should be used directly over the skin graft.
4 - should be used at the donor site to promote faster healing.
PREFERRED RESPONSE: 1 - will likely improve incorporation of the
graft.

DISCUSSION
NPWT increases wound perfusion. The dressing may help decrease risk for
wound infection, but will not do so by removing bacteria. It also helps to
prevent wound contracture to improve the likelihood of primary wound
closure. NPWT can help to improve tissue edema and will not elevate
compartment pressure.
Hemorrhage is the most common major complication associated with
NPWT. This risk is highest when NPWT is used in areas of major vessels
and vessels that have been ligated and for patients undergoing
anticoagulation therapy. Specialized white polyvinyl alcohol sponges are
available to prevent adherence to vessels, exposed nerves, or exposed bone.
NPWT should not be used directly over exposed major vessels. If major
bleeding occurs, a return to the operating room for wound exploration is
recommended.
Incisional NPWT is an effective treatment for persistent serous drainage.
Wounds that drain persistently seal more quickly and pose lower risk for
infection when incisional NPWT is used vs compressive dressings.
Incisional NPWT has also demonstrated benefit when used on high-risk
postsurgical wounds of the tibial plateau, pilon, and calcaneus. It has not
been shown to contribute to increased risk for wound dehiscence or
hematoma.
22

Musculoskeletal Trauma Scored and Recorded Self-Assessment Examination 2015

NPWT stabilizes the soft-tissue environment and does not necessitate


frequent dressing changes. Despite this benefit, a delay of flap coverage after
NPWT still poses higher risk for infection than early coverage.
Consequently, flaps should not be delayed for long. NPWT promotes the
formation of granulation tissue and can be used over exposed bone, but it
would not be expected to form granulation tissue over an exposed fracture
site or hardware or promote fracture healing.
NPWT provides an excellent bolster for a skin graft and improves skin graft
incorporation. It needs to be applied with nonadherent dressings to prevent
adherence to the skin graft. NPWT is generally not used at skin grafting
donor sites.

23

Musculoskeletal Trauma Scored and Recorded Self-Assessment Examination 2015

Musculoskeletal Trauma Scored and Recorded Self-Assessment


Examination 2015
Question 21 of 101
Preventing "missed" femoral neck fractures associated with ipsilateral
femoral shaft fractures is best achieved with
1 - an examination.
2 - dedicated anteroposterior and lateral hip radiographs.
3 - thin-cut pelvic CT images with coronal and sagittal reconstructions.
4 - MRI.
PREFERRED RESPONSE: 3 - thin-cut pelvic CT images with coronal
and sagittal reconstructions.

DISCUSSION
Ipsilateral femoral neck and shaft fractures occur in up to 6% of femur
fractures. A femoral neck fracture is often vertical and nondisplaced. A high
degree of suspicion is necessary to avoid "missed" femoral neck fractures in
patients with this condition. Although an examination and dedicated hip
radiographs help to avoid missed injuries, a significant decrease in missed
injuries has been described with the use of thin-cut pelvic CT images. In
patients who undergo trauma, a pelvic CT scan is often performed to assess
for associated injuries and is easily reviewed to examine the femoral neck.
Although MRI is advocated to identify isolated occult femoral neck
fractures, CT has been described as the method of choice with which to
identify ipsilateral femoral neck and shaft fractures in the trauma population.
Currently, no literature supports the use of MRI in this population.

24

Musculoskeletal Trauma Scored and Recorded Self-Assessment Examination 2015

Musculoskeletal Trauma Scored and Recorded Self-Assessment


Examination 2015

Figure 22
CLINICAL SITUATION FOR QUESTIONS 22 THROUGH 25
Figure 22 is the anteroposterior radiograph of a 44-year-old firefighter who
falls from his road bike and sustains a closed midshaft clavicle fracture. He
chooses surgical treatment with open reduction and internal fixation (ORIF).
Question 22 of 101
What is the most common complication of nonsurgical treatment for this
injury?
1 - Anterior chest wall numbness
2 - Symptomatic malunion
3 - Nonunion
4 - Pneumothorax
PREFERRED RESPONSE: 3 - Nonunion

25

Musculoskeletal Trauma Scored and Recorded Self-Assessment Examination 2015

Question 23 of 101
Which variable is a risk factor for nonunion of displaced clavicle fractures?
1 - Adolescence
2 - Displacement exceeding 100%
3 - Transverse fracture
4 - Male gender
PREFERRED RESPONSE: 2 - Displacement exceeding 100%

Question 24 of 101
The patient decides to undergo surgery with open reduction and plate
fixation. What is the most common reason for revision surgery after plate
fixation of a clavicle fracture?
1 - Supraclavicular nerve entrapment
2 - Symptomatic malunion
3 - Nonunion
4 - Hardware irritation/prominence
PREFERRED RESPONSE: 4 - Hardware irritation/prominence

26

Musculoskeletal Trauma Scored and Recorded Self-Assessment Examination 2015

Question 25 of 101
Which structure(s) is/are most at risk with surgical treatment of displaced
clavicle fractures with ORIF?
1 - Subclavian artery
2 - Subclavian vein
3 - Brachial plexus
4 - Supraclavicular nerves
PREFERRED RESPONSE: 4 - Supraclavicular nerves

DISCUSSION
Complications associated with nonsurgical treatment of displaced midshaft
clavicle fractures are uncommon. Although intrathoracic and local vascular
complications have been reported with clavicle fracture, subclavian artery
aneurysm and pneumothorax are rare. Malunion to some degree is inevitable
with nonsurgical treatment of displaced clavicle fractures, but only about 9%
of patients develop symptomatic malunion. Nonunion occurs in about 15%
of patients.
Previously identified risk factors for nonunion of clavicle fractures include
female gender, displacement exceeding 100%, comminution, and advanced
age. Research demonstrates the strongest risk factors are smoking,
comminution, and fracture displacement. Rate of nonunion in 1 study was
approximately 13%. Murray and associates showed that by estimating the
risk of nonunion using their model and operating only on fractures with at
least a 40% chance of nonunion, they would only need to operate on 1.7
patients to prevent 1 nonunion (decreased from 7.5 procedures per nonunion
if operating on all displaced midshaft fractures). This data could potentially
be used to limit unnecessary procedures and decrease costs associated with
treatment of clavicle fractures.
Hardware removal is the most common reason for revision surgery.
Symptomatic malunion and supraclavicular nerve entrapment are rare after
surgery. Nonunion is uncommon (in fewer than 2% of cases). The main
27

Musculoskeletal Trauma Scored and Recorded Self-Assessment Examination 2015

reason for revision surgery is hardware removal to address local


irritation/prominent hardware or infection.
An anatomical study demonstrated that in 97% of clavicles, 2 to 3 branches
of the supraclavicular nerve were crossing the clavicle with wide location
variability in the zone in which most clavicle fractures occur and surgery
would take place. The subclavian vein and artery and brachial are rarely
injured, although there are case reports of injury to all either by the displaced
fracture fragments or errant hardware.

28

Musculoskeletal Trauma Scored and Recorded Self-Assessment Examination 2015

Musculoskeletal Trauma Scored and Recorded Self-Assessment


Examination 2015

Figure 26a

Figure 26b

Figure 26c

Figure 27

CLINICAL SITUATION FOR QUESTIONS 26 AND 27


Figures 26a through 26c are the radiographs of a 50-year-old athlete who
sustained an injury to his right foot; the foot was plantar flexed and another
player landed on the posterior aspect of his heel. After sustaining the injury
he was unable to bear weight, and 3 days later he was seen in the emergency
department because of persistent pain and tenderness over his midfoot.
Question 26 of 101
CT images reveal a purely ligamentous injury. Which treatment produces the
best results?
1234-

Open reduction and internal fixation (ORIF) of the fracture


Early fusion of the first and second tarsometatarsal joints
Nonweight-bearing activity for 6 weeks
Weight bearing with a camwalker

PREFERRED RESPONSE: 2 tarsometatarsal joints

Early fusion of the first and second


29

Musculoskeletal Trauma Scored and Recorded Self-Assessment Examination 2015

Question 27 of 101
ORIF of the injury was chosen (as illustrated in Figure 27). Long-term
results may include
1 - improved American Orthopaedic Foot & Ankle Society (AOFAS) scores
as compared to scores obtained following fusion.
2 - pes planovalgus.
3 - persistent pain and arthritis.
4 - hindfoot pain.
PREFERRED RESPONSE: 3 - persistent pain and arthritis.

DISCUSSION
The injury mechanism describes axial loading to a plantar-flexed foot and is
classic for Lisfranc injury. If the initial films are not diagnostic as in this
case, weight-bearing films are a reasonable next step. Radiographic
widening of 2 mm or more between the second metatarsal base and medial
cuneiform (as compared to the other side) is diagnostic; occasionally, a
"fleck" sign (a small bony fragment noted in the Lisfranc joint) may indicate
an avulsion fracture. Clinical signs include plantar ecchymosis, tenderness
over the Lisfranc joint, and an inability to bear weight. Anatomic ORIF or
fusion are the options for treatment, and results for ligamentous injuries are
better when fusion is performed. Better AOFAS scores have been
demonstrated with fusion, and a higher incidence of pain and arthritis have
been noted with fixation. No significant difference has been seen regarding
hardware failure, and hindfoot pain is not a consideration.

30

Musculoskeletal Trauma Scored and Recorded Self-Assessment Examination 2015

Musculoskeletal Trauma Scored and Recorded Self-Assessment


Examination 2015
Question 28 of 101
A 67-year-old right-hand-dominant man who is an avid golfer sustains an
unstable distal radius fracture on his right side. He undergoes a closed
reduction with acceptable alignment. After discussing surgical vs
nonsurgical management and recovery, the patient decides to have surgery.
He made this decision because he was told that his
1 - functional outcome at 1 year would be worse with nonsurgical
management.
2 - radiographs will look better after surgery.
3 - grip strength will be better with surgical intervention.
4 - overall long-term outcome can improve with formal occupational therapy
after surgery.
PREFERRED RESPONSE: 3 - grip strength will be better with surgical
intervention.

DISCUSSION
The optimal treatment of distal radius fractures in elderly patients remains
controversial. Both surgical and nonsurgical management of distal radius
fractures produce identical functional outcomes at 1 year. Although many
patients have better motion early with surgery, only grip strength has been
shown to be significantly better at 1 year. Radiographic outcome has not
been correlated with functional outcome, and complications are also
equivalent. Independent prescribed therapy has been better than formal
occupational therapy for range of motion, but no differences in functional
outcome were seen as assessed by Disabilities of the Arm, Shoulder and
Hand scores.

31

Musculoskeletal Trauma Scored and Recorded Self-Assessment Examination 2015

Musculoskeletal Trauma Scored and Recorded Self-Assessment Examination 2015

Figure 29
Question 29 of 101
Figure 29 is the anteroposterior radiograph of a 60-year-old man who is
involved in a motorcycle collision and airlifted to a trauma center. The
patient is hypotensive and tachycardic upon arrival and fluid resuscitation is
underway. He has a scrotal hematoma and his bilateral lower extremities are
externally rotated. What is the first step in managing this scenario?
1 - Obtain CT images of the pelvis
2 - Angiography
3 - Place external fixation
4 - Apply a pelvic binder
PREFERRED RESPONSE: 4 - Apply a pelvic binder
DISCUSSION
A pelvic binder or sheet can be applied right away to reduce and stabilize the
pelvis more quickly than is possible with an external fixator. Pelvic ring
injuries are associated with a high incidence of mortality mainly because of
the potential for retroperitoneal hemorrhage. A pelvic circumferential
compression device allows for force-controlled circumferential compression.
It can effectively reduce pelvic ring injuries and poses minimal risk for
overcompression and complications. Reduction of external rotation injuries
32

Musculoskeletal Trauma Scored and Recorded Self-Assessment Examination 2015

is comparable to definitive fixation reduction and does not cause significant


overcompression of internal rotation injuries. Angiography is used to assess
persistent hemodynamic instability after initial stabilization of the pelvic
ring with the binder or sheet. CT images should be obtained after initial
resuscitation.

33

Musculoskeletal Trauma Scored and Recorded Self-Assessment Examination 2015

Musculoskeletal Trauma Scored and Recorded Self-Assessment


Examination 2015
Question 30 of 101
An 82-year-old woman falls from a standing height and sustains a proximal
humerus fracture. Which factor is the best predictor of ischemia of the
humeral head?
1 - Fracture pattern involving 4 parts
2 - Humeral head angulation exceeding 45 degrees
3 - Posteromedial calcar length of less than 8 mm attached to the humeral
head
4 - Glenohumeral dislocation
PREFERRED RESPONSE: 3 - Posteromedial calcar length of less than 8
mm attached to the humeral head

DISCUSSION
Humeral head ischemia that occurs following proximal humerus fractures is
closely associated with the amount of posteromedial calcar bone attached to
the humeral head. Fractures that exit within 8 mm of the posteromedial edge
of the head more commonly are ischemic (compared to fractures that have
more than 8 mm of posteromedial calcar still attached). Four-part fracture
patterns are a moderate predictor of humeral head ischemia, with an
accuracy of 0.67. Angulation of the humeral head exceeding 45 degrees also
is a moderate predictor of humeral head perfusion, with an accuracy of 0.62.
Glenohumeral dislocation is a poor predictor of humeral head ischemia.

34

Musculoskeletal Trauma Scored and Recorded Self-Assessment Examination 2015

Musculoskeletal Trauma Scored and Recorded Self-Assessment


Examination 2015
Question 31 of 101
An atypical bisphosphonate-associated femur fracture would show which
features?
1. Lateral cortical thickening at the subtrochanteric region with a
fracture line extending to the medial side.
2. Lateral cortical thickening at the supracondylar region with a fracture
line extending to the medial side.
3. Stress fracture of the femoral neck.
4. Reverse obliquity intertrochanteric femur fracture.
PREFERRED RESPONSE: 1 - Lateral cortical thickening at the
subtrochanteric region with a fracture line extending to the medial side

DISCUSSION
Patients sustaining atypical femur fractures have classic radiographic
findings including medial beaking, lateral cortical thickening, and transverse
or short oblique proximal (subtrochanteric) femur fracture.

35

Musculoskeletal Trauma Scored and Recorded Self-Assessment Examination 2015

Musculoskeletal Trauma Scored and Recorded Self-Assessment Examination 2015

RESPONSES FOR QUESTIONS 32 THROUGH 35


1. Open reduction and internal fixation with a proximal humerus locking
plate.
2. Nonsurgical treatment.
3. Arthroplasty
Match the appropriate treatment listed with the clinical scenario described.
Question 32 of 101
A 78-year-old right-hand-dominant woman who lives independently falls
down the stairs at her home. She has an isolated injury to her left shoulder
and a history of hypertension and atrial fibrillation. Imaging reveals a
proximal humerus fracture with a displaced fracture splitting the humeral
head and a large displaced greater tuberosity fragment.
1. Open reduction and internal fixation with a proximal humerus
locking plate.
2. Nonsurgical treatment.
3. Arthroplasty
PREFERRED RESPONSE: 3 - Arthroplasty
Question 33 of 101
A 72-year-old right-hand-dominant woman sustains an isolated injury to her
right shoulder after a fall while walking her dog. She lives independently
and has a history of hypercholesterolemia. Her activities include walking,
aerobics, and yoga. Imaging reveals a proximal humerus fracture with
fracture of the surgical neck that is displaced 2 cm.
1 - Open reduction and internal fixation with a proximal humerus locking plate
2 - Nonsurgical treatment
3 - Arthroplasty
PREFERRED RESPONSE: 1 - Open reduction and internal fixation with a
proximal humerus locking plate

36

Musculoskeletal Trauma Scored and Recorded Self-Assessment Examination 2015

Question 34 of 101
An 81-year-old left-hand-dominant woman who lives independently has a
slip-and-fall accident while shopping. She has an isolated injury to her left
shoulder and a history of coronary artery disease and hypertension. Imaging
reveals a proximal humerus fracture with 50% translation at the surgical
neck.
1 - Open reduction and internal fixation with a proximal humerus locking
plate
2 - Nonsurgical treatment
3 - Arthroplasty
PREFERRED RESPONSE: 2 - Nonsurgical treatment

Question 35 of 101
An 80-year-old right-hand-dominant woman who lives independently falls
in her home. She has an isolated injury to her right shoulder and a history of
a total hip replacement for a femoral neck fracture (3 years prior). She has
had a prior failed rotator cuff repair. Her daily activities include volunteering
at her church and caring for her grandchildren. Imaging reveals a displaced
proximal humerus fracture with comminution and 50% translation with
varus angulation at the surgical neck. She also has displacement and
comminution of the greater tuberosity.
1 - Open reduction and internal fixation with a proximal humerus locking
plate
2 - Nonsurgical treatment
3 - Arthroplasty
PREFERRED RESPONSE: 3 - Arthroplasty
DISCUSSION
Treatment of proximal humerus fractures in elderly patients is controversial
and requires consideration of the patient's functional demands and fracture
characteristics. The majority of fractures can be treated nonsurgically.
37

Musculoskeletal Trauma Scored and Recorded Self-Assessment Examination 2015

Nonsurgically treated fractures should be briefly immobilized before


beginning pendulum exercises and elbow range of motion.
Some patients are surgical candidates based upon functional demands and
degree of displacement. Fractures that are reconstructible can be
successfully treated with reduction and fixation or intramedullary nailing. If
there is tuberosity involvement, plate fixation is preferable to intramedullary
nailing. Factors that make fixation challenging and vulnerable to failure
include poor bone quality and significant varus alignment. Relative
indications for arthroplasty, especially in patients with poor bone quality,
include initial varus alignment, head-splitting fractures, and 4-part fractures.
Traditionally, hemiarthroplasty has produced reliable pain relief and
unreliable function because of the difficulty associated with reconstruction
of the tuberosities to restore rotator cuff function. Reverse shoulder
arthroplasty may be a better option for patients who are arthroplasty
candidates who have tuberosities that will not be reliably reconstructed or
for those who have a pre-existing rotator cuff deficiency.

38

Musculoskeletal Trauma Scored and Recorded Self-Assessment Examination 2015

Musculoskeletal Trauma Scored and Recorded Self-Assessment


Examination 2015

Figure 36
Question 36 of 101
Two femoral shaft fractures are shown in Figure 36. Each is fixed identically
with the same intramedullary nail and interlocking screws. The fracture gap
strain is higher in
1 - A.
2 - B.
3 - neither; the strain is identical in A and B.
4 - neither; the strain is dependent on femur length.
PREFERRED RESPONSE: 1 - A.

DISCUSSION
Fracture gap strain is defined as deformation of granulation tissue within the
fracture gap when a given force is applied. Normal strain is the change in
length (? l) divided by the original length (l) when a given load is applied.
39

Musculoskeletal Trauma Scored and Recorded Self-Assessment Examination 2015

The amount of deformation that a tissue can tolerate while functioning varies
greatly. Intact bone has a normal strain tolerance of 2% (before it fractures),
whereas granulation tissue has a strain tolerance of 100%. Bony bridging
between the distal and proximal callus can only occur when local strain (ie,
deformation) is less severe than the forming bone can tolerate. Therefore,
treatment of fractures must optimize the strain environment to enable
healing.
Comminution, as shown in B, results in distribution of the motion between
multiple fracture fragments. As a result, each fracture gap experiences less
motion and strain is decreased. In simple fracture patterns as shown in A,
small amounts of motion or even a small fracture gap results in a high-strain
environment. Strain is dependent upon the length of the fracture gap but not
on the length of the bone.

40

Musculoskeletal Trauma Scored and Recorded Self-Assessment Examination 2015

Musculoskeletal Trauma Scored and Recorded Self-Assessment Examination 2015

Figure 37
Question 37 of 101
Figure 37 is the radiograph of a 31-year-old woman who has acute right hip
pain after a fall. The treatment variables that are most important to maximize
clinical outcome are
1 - timing of fixation and capsulotomy.
2 - timing of fixation and choice of implants.
3 - quality of reduction and fixation.
4 - choice of open reduction and capsulotomy.
PREFERRED RESPONSE: 3 - quality of reduction and fixation.
DISCUSSION
Femoral neck fractures are potentially devastating injuries for physiologically
young patients. Studies have demonstrated that the timing of fixation is not as
critical to outcome or to avascular necrosis prevention as other factors.
Experimental evidence supports capsulotomy to improve femoral head blood flow.
Relative biomechanical advantages are associated with different implants;
however, a surgeon can obtain good fixation with a variety of devices. For a
physiologically young patient, an open reduction is often required to obtain the
desired anatomic reduction; however, if the desired result can be achieved with
closed reduction, open reduction is not required. Anatomic reduction of the
fracture and biomechanically sound fixation consistently yield optimal results.
41

Musculoskeletal Trauma Scored and Recorded Self-Assessment Examination 2015

Musculoskeletal Trauma Scored and Recorded Self-Assessment


Examination 2015

Figure 38a

Figure 38b

Figure 38c

Question 38 of 101
A 55-year-old man fell off a bicycle and sustained the injury shown in
Figures 38a through 38c. Which fracture pattern best describes this injury?
1 - Anterior column posterior hemitransverse
2 - Anterior column
3 - Anterior wall
4 - Associated both-column
PREFERRED RESPONSE: 2 - Anterior column

DISCUSSION
This is an anterior column fracture with dome impaction. The obturator
oblique view and both CT images show disruption of the anterior column.
Both CT images also reveal an intact posterior column, which eliminates
anterior column posterior hemitransverse and associated both-column
fracture types as correct responses. An anterior wall fracture would not
extend up into the ilium.
42

Musculoskeletal Trauma Scored and Recorded Self-Assessment Examination 2015

Musculoskeletal Trauma Scored and Recorded Self-Assessment Examination 2015

Figure 39
CLINICAL SITUATION FOR QUESTIONS 39 THROUGH 41
Figure 39 is the standing radiograph of a 20-year-old college student who
injures his foot while playing intramural football. Initial radiograph findings
are reportedly normal, but 1 week after injury he still cannot bear weight.
You see him in the clinic and note swelling of his foot and plantar
ecchymosis.
Question 39 of 101
What is the strongest structure supporting the tarsometatarsal (TMT)
complex of the midfoot?
1234-

Oblique interosseous ligament


Deep band of the plantar oblique ligament
Dorsal oblique ligament
First TMT ligament

PREFERRED RESPONSE: 1 -

Oblique interosseous ligament

43

Musculoskeletal Trauma Scored and Recorded Self-Assessment Examination 2015

Question 40 of 101
What radiographic finding is consistent with a Lisfranc injury?
1 - Dorsal and plantar aspects of the metatarsals (MTs) correspond with the
cuneiforms and cuboid on the lateral view.
2 - The medial border of the second MT is aligned with the medial border of
the middle cuneiform on the anteroposterior view.
3 - The medial border of the fourth MT is aligned with the medial border of
the cuboid on the oblique view.
4 - Diastasis between the first and second MT is 3.5 mm.
PREFERRED RESPONSE: 4 - Diastasis between the first and second MT
is 3.5 mm.

Question 41 of 101
Primary arthrodesis is associated with which outcome when compared to
outcomes associated with open reduction and internal fixation (ORIF)
without arthrodesis?
1234-

Decreased secondary surgeries


Increased pain
Increased risk for infection
Poorer function at 2-year follow-up

PREFERRED RESPONSE: 1 -

Decreased secondary surgeries

DISCUSSION
There are longitudinal, oblique, and transverse ligaments at the TMT
complex that are further defined by their location as dorsal, interosseous, or
plantar. There are 3 ligaments between the medial cuneiform and the second
MT base, the most important of which is the oblique interosseous ligament,
which is also known as the Lisfranc ligament. Plantar and dorsal oblique
ligaments contribute to stability to a lesser degree. The dorsal ligaments are
weakest and may be the first to fail in a Lisfranc injury.
44

Musculoskeletal Trauma Scored and Recorded Self-Assessment Examination 2015

The second MT base should be aligned with the middle cuneiform at the
medial borders, and the fourth MT base should be aligned with the cuboid at
the medial borders on the oblique view. The dorsal and plantar aspects of the
MTs should align with the cuneiforms/cuboid on the lateral view. Any
malalignment should raise the suspicion of a Lisfranc injury. Diastasis
between the second MT and the first MT/medial cuneiform complex of more
than 2 mm indicates injury, as does TMT joint subluxation of 2 mm more
than seen on the uninjured contralateral side. Diastasis between the first and
second MT up to 2.7 mm can be normal. Another radiographic sign of injury
includes avulsion fracture of the second MT base or medial cuneiform.
Additional imaging studies that may be helpful in identifying subtle injuries
include weight-bearing radiographs and CT or MRI images.
Two prospective randomized studies compared primary fusion with ORIF
Lisfranc injuries. The second study included fracture-dislocations, whereas
the first looked at primarily ligamentous injuries. Results conflicted with an
earlier study demonstrating improved results (less pain, better function) with
primary fusion, while a more recent study showed no difference. Neither
study showed worse results with primary fusion, and the rate of secondary
surgery was more common in the ORIF group (salvage arthrodesis or
hardware removal).

45

Musculoskeletal Trauma Scored and Recorded Self-Assessment Examination 2015

Musculoskeletal Trauma Scored and Recorded Self-Assessment Examination 2015

Figure 42a

Figure 42b

Question 42 of 101
Which ligament attaches to the bony fragment identified by the CT image
arrows in Figures 42a and 42b?
1 - Posterior tibiotalar ligament
2 - Posterior-inferior tibiofibular ligament (PITFL)
3 - Interosseous ligament (IOL)
4 - Anterior-inferior tibiofibular ligament (AITFL) ?
PREFERRED RESPONSE: 2 - Posterior-inferior tibiofibular ligament
(PITFL)
DISCUSSION
The distal tibiofibular syndesmosis is a ligamentous complex that consists of the
AITFL, PITFL, intertransverse ligament (ITL), and IOL. The PITFL originates on
the posterior inferior aspect of the tibia (Volkmann tubercle) and inserts on the
lateral malleolus. The AITFL originates on the anterolateral aspect of the tibia
(Chaput tubercle) and inserts on the distal anterior aspect of the fibula (Wagstaffe
tubercle). The ITL is a group of fibers running transversely just inferior to the
PITFL. As a group, these structures maintain the appropriate tibial plafond and
talus relationship throughout physiologic range of motion.
46

Musculoskeletal Trauma Scored and Recorded Self-Assessment Examination 2015

Musculoskeletal Trauma Scored and Recorded Self-Assessment Examination 2015

Figure 43a

Figure 43b

CLINICAL SITUATION FOR QUESTIONS 43 THROUGH 46


A 46-year-old healthy right-hand-dominant man falls and sustains the injury
shown in Figures 43a and 43b.
Question 43 of 101
What is the treatment of choice?
1234-

Nonsurgical treatment
Open reduction and internal fixation (ORIF)
Hemiarthroplasty
Total shoulder arthroplasty (TSA)

PREFERRED RESPONSE: 2 (ORIF)

Open reduction and internal fixation

47

Musculoskeletal Trauma Scored and Recorded Self-Assessment Examination 2015

Question 44 of 101
If the patient undergoes ORIF, which strategy is essential to minimize
fixation failure?
1 - Use of all locking screws
2 - Use of cancellous allograft for defect management
3 - Achieving at least 3 points of fixation in the humeral head
4 - Restoration of medial cortical support
PREFERRED RESPONSE: 4 - Restoration of medial cortical support

Question 45 of 101
A similar fracture is treated with ORIF and a locking plate for an active 73year-old right-hand-dominant woman. Which patient characteristic is most
likely to contribute to possible fixation failure?
1234-

Hand dominance
Level of activity
Osteoporosis
Rotator cuff incompetence

PREFERRED RESPONSE: 3 -

Osteoporosis

48

Musculoskeletal Trauma Scored and Recorded Self-Assessment Examination 2015

Question 46 of 101
The 73-year-old patient undergoes shoulder hemiarthroplasty. What is a risk
factor for a poor outcome?
1 - Tuberosity nonunion
2 - Hand dominance
3 - Female gender
4 - BMI higher than 30
PREFERRED RESPONSE: 1 - Tuberosity nonunion

DISCUSSION
Surgical treatment is favored for young, active patients with displaced
proximal humerus fractures. Nonsurgical treatment is favored to treat
fractures with minimal displacement among low-demand elderly patients.
When ORIF is used, a number of strategies are employed to prevent failure,
including restoration of medial cortical support (medial calcar),
incorporation of the rotator cuff into the construct, and placement of screws
of adequate length to gain purchase in the subchondral bone of the humeral
head. Intramedullary allograft is not routinely required but is useful when
dealing with osteoporotic bone. Cancellous allograft has not been shown to
prevent failure. Varus collapse and failure of fixation are more prevalent in
patients with osteoporotic bone, and, in these cases, strategies for
supplemental fixation are advisable. In cases of severe osteoporosis,
comminution, or poor bone quality, shoulder arthroplasty may be a better
choice. Without a functioning rotator cuff, as would happen with a
tuberosity nonunion, outcomes after shoulder hemiarthroplasty and TSA are
poor.

49

Musculoskeletal Trauma Scored and Recorded Self-Assessment Examination 2015

Musculoskeletal Trauma Scored and Recorded Self-Assessment


Examination 2015

Figure 49a

Figure 49b

Figure 50a

Figure 49c

Figure 50b
50

Musculoskeletal Trauma Scored and Recorded Self-Assessment Examination 2015

RESPONSES FOR QUESTIONS 47 THROUGH 50


1 - Stress distribution
2 - Stress concentration
For each pattern detailed or depicted, select the appropriate condition.
Question 47 of 101
A simple fracture pattern that is nonanatomically reduced with a 3-mm gap
and treated with an 8-hole locking plate with 4 bicortical locking screws
placed on each side of the fracture
1 - Stress distribution
2 - Stress concentration
PREFERRED RESPONSE: 2 - Stress concentration

Question 48 of 101
A multifragmentary fracture pattern that is bridge plated to restore length
and alignment and treated with a 12-hole locking plate with 4 bicortical
locking screws placed on each side of the fracture
1 - Stress distribution
2 - Stress concentration
PREFERRED RESPONSE: 1 - Stress distribution

Question 49 of 101
Figures 49a through 49c
1 - Stress distribution
2 - Stress concentration
PREFERRED RESPONSE: 2 - Stress concentration
51

Musculoskeletal Trauma Scored and Recorded Self-Assessment Examination 2015

Question 50 of 101
Figures 50a and 50b
1 - Stress distribution
2 - Stress concentration
PREFERRED RESPONSE: 1 - Stress distribution

Figure 50d

Figure 50c
DISCUSSION

When comparing stress distribution and stress concentration, the focus is


primarily on the implant. Stress is equal to force divided by the area over
which that force is distributed. When the area is small, concentration of
stress occurs. When the area is large, distribution of stress occurs. The
practical importance is most easily understood via an analogy (Figures 50c
and 50d). Consider a ruler. If the goal were to break the ruler, placing your
thumbs close together would be a logical choice. This hand position
concentrates the forces over a small area (stress concentration). Now
imagine that the ruler is a bone and your thumbs are screws placed on each
side of a fracture. If a bending load is applied, the same small area of the
plate is cycled. Metal can sustain a limited number of cycles before fatigue
failure occurs. If the bone does not heal before this time, construct failure
ensues. In a scenario in which prolonged healing times are expected, leaving
a larger segment of the plate unsupported (ie, moving the center screws
further away from each other) would distribute implant stress. This must be
balanced with the goal of stability and the basic science of bone healing.

52

Musculoskeletal Trauma Scored and Recorded Self-Assessment Examination 2015

Musculoskeletal Trauma Scored and Recorded Self-Assessment


Examination 2015

Figure 51a

Figure 52b

Figure 51b

Figure 52c

Figure 52a

Figure 52d

CLINICAL SITUATION FOR QUESTIONS 51 THROUGH 53


Figures 51a and 51b are the radiographs of a 55-year-old man who was
involved in a motor vehicle collision. The patient has pain and deformity of
his right knee. Examination reveals crepitus and swelling about the knee
53

Musculoskeletal Trauma Scored and Recorded Self-Assessment Examination 2015

with gross motion of the distal femur. There is an 8-cm lateral open wound
with exposed bone and gross contamination.
Question 51 of 101
Immediate surgical treatment should consist of irrigation and debridement of
the fracture and
1234-

bridging external fixation.


a retrograde intramedullary nail.
a tibial traction pin.
open reduction and internal fixation (ORIF) with plate fixation.

PREFERRED RESPONSE: 1 -

bridging external fixation.

Question 52 of 101
The patient undergoes ORIF as shown in Figures 52a and 52b. Three months
later, he develops a deformity and pain. Radiographs are shown in Figures
52c and 52d. Early hardware failure in the management of distal femur
fractures has been linked to
1234-

the use of nonlocking screws in the proximal fragment.


the length of the plate used.
distal placement of the plate.
comminution of the fracture.

PREFERRED RESPONSE: 2 -

the length of the plate used.

54

Musculoskeletal Trauma Scored and Recorded Self-Assessment Examination 2015

Question 53 of 101
When applying a locking plate to the lateral aspect of the distal femur,
medial translation of the distal femur occurs with respect to the diaphysis
("golf-club deformity"). This deformity was created by
1234-

placement of the plate too posterior.


placement of the plate too proximal.
placement of the plate too anteriorly on the shaft.
placement of the plate too anteriorly on the condyles.

PREFERRED RESPONSE: 1 -

placement of the plate too posterior.

DISCUSSION
This patient should undergo immediate irrigation and debridement of the
fracture. Secondary to gross contamination, there is concern for initial
definitive treatment. ?
Because of the contamination, temporary stabilization will allow for better
soft-tissue management and a second look prior to definitive internal
fixation. Tibial traction pin placement for skeletal traction is less than ideal
because it confines patients to bed rest. With external fixation, a patient can
mobilize.
Obtaining CT images prior to ORIF will aid in preoperative planning.
Locked plating of supracondylar distal femur fractures has not been without
complications. In a large study of patients undergoing surgical fixation, it
was found that a key failure factor was plate length. A plate longer than 9
holes (shaft) that allows for at least 8 holes proximal to the fracture is ideal.
Other risk factors that led to implant failure in this study were obesity, open
fractures, smoking, and younger age. There has not been an association with
early failure using nonlocked screws or the degree of comminution. Both
may be factors in long-term failure if there is delayed healing or nonunion
development. ?
The golf-club deformity has been a long-standing problem in the
management of distal femur fractures when a plate is applied too posteriorly.
This was true when 95-degree dynamic condylar plates or blade plates were
55

Musculoskeletal Trauma Scored and Recorded Self-Assessment Examination 2015

used, and this still holds true for locking plates. Distal placement of the plate
also leads to this deformity because in both situations medialization of the
condyles occurs. Placing the plate too anterior on the shaft can lead to
compromised fixation and early failure, whereas placement anterior on the
condyles can lead to hardware pain or intra-articular screw penetration into
the patella-femoral joint. Proximal placement of the plate would not result in
the deformity and is not a common problem because of the contour of the
plate. If the plate were applied too proximal, the condyles would be
lateralized and/or insufficient points of fixation could occur.

56

Musculoskeletal Trauma Scored and Recorded Self-Assessment Examination 2015

Musculoskeletal Trauma Scored and Recorded Self-Assessment


Examination 2015

Figure 54a

Figure 54b

Question 54 of 101
A 53-year-old man is involved in a motor vehicle collision and sustains the
closed distal femur fracture seen in Figures 54a and 54b. A precontoured
distal femoral locking plate is selected for fixation. A locking construct
should be used to
1 - make the construct as rigid as possible and minimize strain to promote
primary bone healing.
2 - make the construct as rigid as possible and provide a high-strain
environment to promote primary bone healing.
3 - provide a fixed-angle construct and bridge the area of comminution to
minimize strain and promote secondary bone healing.
4 - provide a fixed-angle construct and bridge the area of comminution to
provide a high-strain environment and promote secondary bone healing.
PREFERRED RESPONSE: 3 - provide a fixed-angle construct and bridge
the area of comminution to minimize strain and promote secondary bone
healing.

57

Musculoskeletal Trauma Scored and Recorded Self-Assessment Examination 2015

DISCUSSION
This patient has a comminuted distal femur fracture. A fixed-angle device
such as a locking plate is preferred to confer angular stability to the construct
and prevent varus collapse. The strategy to promote union of the fracture is
to provide a low-strain environment to allow bone healing. Strain is
determined by the amount of motion over the length of a fracture. In the case
of a noncomminuted fracture, the fracture surfaces can be compressed and
rigid fixation applied to abolish strain and promote primary bone healing
without callus. In the case of a comminuted fracture, the preferred fixation
strategy focuses on distributing motion along the length of the fracture to
provide a low-strain environment that will promote secondary bone healing
and callus formation.

58

Musculoskeletal Trauma Scored and Recorded Self-Assessment Examination 2015

Musculoskeletal Trauma Scored and Recorded Self-Assessment


Examination 2015
Question 55 of 101
An 83-year-old right-hand-dominant woman sustains a displaced right extraarticular distal radius fracture and is treated with closed reduction and
casting. At her 4-week follow-up visit, radiographs demonstrate a volar tilt
of -5 degrees and 4 mm of positive ulnar variance. Which treatment is
recommended?
1 - No additional reduction and continued treatment in the cast
2 - Repeat closed reduction and cast application
3 - Closed reduction and percutaneous skeletal fixation
4 - Open reduction and internal fixation
PREFERRED RESPONSE: 1 - No additional reduction and continued
treatment in the cast

DISCUSSION
Studies demonstrate that surgical treatment of distal radius fractures in
elderly people does not result in improved outcomes. Although nonsurgical
treatment resulted in worse radiographic findings for this patient, these
findings did not translate into worse functional outcomes.

59

Musculoskeletal Trauma Scored and Recorded Self-Assessment Examination 2015

Musculoskeletal Trauma Scored and Recorded Self-Assessment


Examination 2015

Figure 56
Question 56 of 101
Figure 56 is the radiograph of a 62-year-old noninsulin-dependent woman
with diabetes who twisted her ankle while walking and felt a pop. At the
emergency department she describes heel pain. What is the best course of
action?
1 - Protected weight-bearing activity for 6 weeks
2 - Closed reduction and cast application
3 - Urgent open reduction and internal fixation
4 - Excision of the calcaneal tuberosity
PREFERRED RESPONSE: 3 - Urgent open reduction and internal fixation
DISCUSSION
The radiograph reveals a displaced calcaneal tuberosity fracture.
Displacement of a large tuberosity fragment necessitates urgent fracture
reduction and stabilization. Delayed reduction results in compromise of the
skin and soft tissues at the posterior heel. This injury occurs frequently in
patients with diabetes. Protected weight-bearing activity does not address the
displaced fragment or the threatened skin. Closed reduction, if possible, will
not maintain the tuberosity fragment in a reduced position and will likely
result in redisplacement. The fragment is large enough that it may be fixed
and not excised. The Achilles tendon inserts on the displaced tuberosity
fragment, so tuberosity reduction and fixation is necessary to achieve proper
Achilles function.
60

Musculoskeletal Trauma Scored and Recorded Self-Assessment Examination 2015

Musculoskeletal Trauma Scored and Recorded Self-Assessment


Examination 2015
RESPONSES FOR QUESTIONS 57 THROUGH 59
1 - High strain
2 - Low strain
For each fracture detailed, select the appropriate description.
Question 57 of 101
A simple fracture pattern that is anatomically reduced and compressed and
treated with an 8-hole conventional plate with 4 bicortical conventional
screws placed on each side of the fracture
1 - High strain
2 - Low strain
PREFERRED RESPONSE: 2 - Low strain

Question 58 of 101
A multifragmentary fracture pattern that is bridge plated, restoring length
and alignment, and treated with a 12-hole locking plate with 4 bicortical
locking screws placed on each side of the fracture
1 - High strain
2 - Low strain
PREFERRED RESPONSE: 2 - Low strain

61

Musculoskeletal Trauma Scored and Recorded Self-Assessment Examination 2015

Question 59 of 101
A transverse humeral shaft fracture that occurs between a stiff arthritic
shoulder joint; a stiff, arthritic elbow joint is treated nonsurgically in a
hanging-arm cast
1 - High strain
2 - Low strain
PREFERRED RESPONSE: 1 - High strain

DISCUSSION
In 1977, Perren and Cordey penned a German manuscript that first described
an interpretation of mechanical influences on tissue differentiation. This
became known as the Strain Theory of Perren. In 1980, a second manuscript
by the same authors was published in English. Within this manuscript,
Perren wrote, "These thoughts about the mechanical influences on tissue
differentiation are not intended as conclusive evidence since precise data are
still not available, but we hope that they will stimulate thought and provide a
basis for discussion." More than 30 years later, these thoughts continue to
stimulate discussion and research on cell mechanotransduction. This theory
is still being manipulated in surgical theatres all around the world in an
attempt to more consistently achieve fracture healing. Strain is a magnitude
of deformation. As typically defined, it is the change in dimension of a
deformed object during loading divided by its original dimension. This is
difficult to work with intraoperatively. The fraction below illustrates a
simpler way to regard this concept:
Strain = Magnitude of displacement between fragments during loading /
Total resting distance between fragments after stabilization
By remembering that low strain generally leads to bone formation and
healing, it is possible to manipulate this fraction intraoperatively to achieve
success. When a simple fracture pattern is anatomically reduced and
compressed, then the total resting distance between fragments after
stabilization approaches 0. This means the numerator must be near 0 to
achieve a low-strain environment. This is what occurs in absolute stability
62

Musculoskeletal Trauma Scored and Recorded Self-Assessment Examination 2015

(no motion between fracture fragments under physiologic load) and primary
bone healing occurs. When a multifragmentary fracture pattern is treated
with bridge plating, the total resting distance between fragments after
stabilization is a larger number (consider the additive distance between the
different fragments). In this case, the numerator can be larger to achieve a
low-strain environment. This is what happens in relative stability (controlled
motion between fracture fragments under physiologic load). Secondary bone
healing occurs. Now consider the third scenario: a simple fracture pattern
that is fixed with a small gap. The total resting distance is still a small
number. Based on the theory, eliminating motion by creating a stiff construct
should lead to healing, but it does not. Creating absolute stability with a gap
means that primary bone healing cannot occur (because cutting cones cannot
cross the gap) and secondary bone healing cannot occur (because there is not
enough motion to induce callus formation). This is where the strain theory
breaks down and how many nonunions occur. In the fourth scenario, a highstrain environment is present and commonly leads to a nonunion (as
predicted by the theory). The simple fracture pattern is too mobile, and
nonfunctional callus often occurs.

63

Musculoskeletal Trauma Scored and Recorded Self-Assessment Examination 2015

Musculoskeletal Trauma Scored and Recorded Self-Assessment


Examination 2015

Figure 60
Question 60 of 101
The condition seen in Figure 60 is attributable to
1 - improper nail placement.
2 - wrong implant choice.
3 - patient noncompliance.
4 - radius of the curvature of the implant.
PREFERRED RESPONSE: 4 - radius of the curvature of the implant.

DISCUSSION
Cephallomedullary implants for treatment of proximal femur fractures have
gained in popularity over the last decade. Although these implants have
improved outcomes for certain fracture types, multiple complications are
associated with this implant. Failure may occur secondary to implant design
(for example, mismatch of curvature of the nail to the femur, which can
result in distal anterior cortical perforation).

64

Musculoskeletal Trauma Scored and Recorded Self-Assessment Examination 2015

Musculoskeletal Trauma Scored and Recorded Self-Assessment


Examination 2015

Figure 61
Question 61 of 101
Figure 61 is the radiograph of a 42-year-old man who falls from a roof and
sustains a right calcaneus fracture. His hindfoot is moderately swollen
without skin wrinkling and the skin is intact and viable. Neurologic
examination findings are normal and the dorsalis pedis pulse is strong and
palpable. What is the best treatment plan at this time?
1 - Immediate open reduction and internal fixation (ORIF) via an extensile
lateral approach.
2 - Casting in a plantar-flexed position for 6 weeks
3 - Splinting with follow-up in 10 to 14 days to check for resolution of
swelling
4 - Splinting with a repeat examination in 1 to 2 days
PREFERRED RESPONSE: 4 - Splinting with a repeat examination in 1 to
2 days
65

Musculoskeletal Trauma Scored and Recorded Self-Assessment Examination 2015

DISCUSSION
This patient has a displaced tuberosity of the calcaneus. A high rate of
posterior skin breakdown is associated with these fracture types. The skin
should be checked within 10 to 14 days when these fractures occur. The skin
is swollen and not acutely at risk, so an immediate ORIF via an extensile
lateral approach is not warranted. Immobilizing the ankle in a plantar-flexed
position can take some tension off the posterior skin with this fracture type
but should not be definitive treatment. Splinting with repeat examination in
1 to 2 days is the preferred response because of the short follow-up for a
repeat skin check. If the skin is at risk when a fracture of this type occurs,
the ankle can be immobilized in plantar flexion to relieve tension on the
skin. Immediate repair with either open or percutaneous techniques may be
necessary if the skin remains at risk.
Question 62 of 101
A 55-year-old active left-hand-dominant woman sustains a minimally
displaced (less than 2 mm displaced) left proximal humerus fracture that
involves fractures of the greater tuberosity and surgical neck. Her activities
include tennis and golf. What is the best treatment for this patient?
1 - Nonsurgical treatment
2 - Percutaneous skeletal fixation
3 - Open reduction and internal fixation
4 - Total shoulder arthroplasty
PREFERRED RESPONSE: 1 - Nonsurgical treatment

DISCUSSION
Literature guiding indications for surgical vs nonsurgical treatment of
proximal humerus fractures is not definitive. Many of the recommendations
are based on older, nonrandomized series. Newer data suggest that surgical
and nonsurgical treatment provide comparable results. Although surgical
treatment is preferred for treatment of displaced fractures, fractures with
minimal displacement are best managed without surgery.

66

Musculoskeletal Trauma Scored and Recorded Self-Assessment Examination 2015

Musculoskeletal Trauma Scored and Recorded Self-Assessment


Examination 2015

Figure 63b
Figure 63c

Figure 63a
Question 63 of 101

Figures 63a through 63c are the radiographs of a 19-year-old woman who
sustained injuries in a motorcycle collision. Which initial treatment will
result in the best functional outcome and lowest pain scores at 2 years?
1 - Open reduction and internal fixation (ORIF)
2 - Closed reduction and percutaneous pinning
3 - Nonsurgical management with closed reduction and casting
4 - Primary arthrodesis of the first and second tarsometatarsal joints
PREFERRED RESPONSE: 4 - Primary arthrodesis of the first and second
tarsometatarsal joints
DISCUSSION
Ligamentous injuries to the tarsometatarsal and intermetatarsal joints are
commonly a result of high-energy mechanisms. These injuries have resulted
in worse outcomes following ORIF than Lisfranc injuries, which involve
fractures. Multiple injury patterns may occur, with some injuries involving
67

Musculoskeletal Trauma Scored and Recorded Self-Assessment Examination 2015

mostly the ligamentous structures. Ligamentous Lisfranc injuries treated


with primary arthrodesis have been shown to result in improved American
Orthopaedic Foot & Ankle Society scores and lower Visual Analog Scale
pain scores at 2-year follow-up than injuries treated with ORIF. ORIF with
either plate or screw fixation has resulted in higher rates of secondary
surgeries and lower functional scores. Nonsurgical management is not
recommended for displaced injuries. Cast placement is recommended for
patients with midfoot sprains with displacement of less than 2 mm.
Nonanatomic reductions have been associated with poor results. Closed
reduction and percutaneous pinning is unlikely to achieve an anatomic
reduction and stable fixation.

68

Musculoskeletal Trauma Scored and Recorded Self-Assessment Examination 2015

Musculoskeletal Trauma Scored and Recorded Self-Assessment


Examination 2015

Figure 64a

Figure 64b

Question 64 of 101
A 36-year-old healthy man was thrown from a horse and sustained the injury
shown in Figures 64a and 64b. His left buttock has a significant hematoma
with a "fluid wave," and the skin overlying the area is anesthetic (diminished
sensation). How should this patient's injuries be treated?
1 - Early open debridement of the hematoma and open reduction and internal
fixation (ORIF) of the fracture with immediate wound closure
2 - Aspiration and culture of the hematoma, delayed debridement of the
hematoma, ORIF, and wound closure
3 - Nonsurgical management of the wound and fracture
4 - Early percutaneous debridement of the hematoma followed by delayed
ORIF
PREFERRED RESPONSE: 4 - Early percutaneous debridement of the
hematoma followed by delayed ORIF

69

Musculoskeletal Trauma Scored and Recorded Self-Assessment Examination 2015

DISCUSSION
The treatment of a pelvic and/or acetabular fracture associated with a MorelLavallee lesion is fraught with complications. The recommendations are to
debride open followed by ORIF, closing only the fascia and leaving the
remaining wound open, or performing a percutaneous debridement followed
by delayed ORIF. Risk for wound breakdown and infection are higher with
other treatment modalities. Cultures are not necessary because they can be
positive, but are generally not the infecting organism if infection ensues. The
posterior wall fracture pattern mandates surgical management, so
nonsurgical management is not appropriate.

70

Musculoskeletal Trauma Scored and Recorded Self-Assessment Examination 2015

Musculoskeletal Trauma Scored and Recorded Self-Assessment


Examination 2015
CLINICAL SITUATION FOR QUESTIONS 65 THROUGH 67
A 25-year-old man with a Gustilo IIIB open tibia fracture is treated with
initial irrigation and debridement. Negative pressure wound therapy
(NPWT) is initiated while awaiting definitive flap coverage. After 48 hours,
the sponge is changed and erythema is noted at the wound edges.
Question 65 of 101
What is the most likely cause of the erythema?
1234-

Acute bacterial infection


Allergic reaction to the sponge material
Mechanical irritation from contact of the sponge with intact skin
Skin necrosis from the original trauma

PREFERRED RESPONSE: 3 sponge with intact skin

Mechanical irritation from contact of the

Question 66 of 101
What is the NPWT mechanism of action?
1234-

Increased blood flow to the wound


Increased antibiotic concentration in the wound
Angiogenic effect of sponge material on the wound bed
Thermal necrosis kills pathogens at the wound edges

PREFERRED RESPONSE: 1 -

Increased blood flow to the wound

71

Musculoskeletal Trauma Scored and Recorded Self-Assessment Examination 2015

Question 67 of 101
In which scenario is application of NPWT contraindicated?
1 - Fasciotomy wound after compartment syndrome
2 - Exposed bone after debridement
3 - Surgical wound that cannot be closed because of tension
4 - Surgical tumor bed after excision while awaiting final pathology and
definitive closure
PREFERRED RESPONSE: 4 - Surgical tumor bed after excision while
awaiting final pathology and definitive closure

DISCUSSION
The most common complication associated with NPWT is a rash on the skin
resulting from contact with a suction sponge. In a 2001 study by Webb and
Schmidt, 2.2% of patients treated with NPWT had a rash that resolved
within 48 hours. To minimize risk for this rash, care should be taken to
avoid overlap of the sponge onto intact skin. If skin overlap is unavoidable, a
lower-pressure setting should be used.
Acute bacterial infection is possible in this scenario, but there would likely
be additional findings such as purulence in the wound itself. Similarly, an
acute allergic reaction could be found within the wound and not just at the
skin edge. Skin irritation from excessive adhesive plastic tension can result
in blistering or shearing avulsion but not skin-edge erythema. Skin necrosis
from the original trauma would result in skin-edge duskiness instead of
erythema.
A study by Timmers and associates showed a statistically significant
increase in microvascular blood flow to the skin under a sponge at
subatmospheric pressures. Angiogenesis occurs in the area of the wound
vacuum from the negative pressure and increased blood flow, not from the
sponge material itself. There has been no indication that antibiotic
concentration is increased in the setting of negative pressure therapy because
antibiotics would be evacuated along with the fluid. NPWT does not result
in increased temperatures that would kill pathogens.
72

Musculoskeletal Trauma Scored and Recorded Self-Assessment Examination 2015

NPWT is contraindicated in the setting of neoplasm because its effect on


tumors is unknown. There is potential for increased angiogenesis in residual
tumor cells, which could lead to recurrence or even metastasis. NPWT has
been used safely and effectively for coverage of open fractures between
initial debridement and definitive coverage. Fasciotomy wounds are
frequently covered with NPWT dressings on a temporary basis with
excellent results. NPWT has been used to temporize wounds with exposed
bone before flap coverage. Wounds with excessive tension frequently can be
closed after short-term coverage with NPWT dressings.

73

Musculoskeletal Trauma Scored and Recorded Self-Assessment Examination 2015

Musculoskeletal Trauma Scored and Recorded Self-Assessment


Examination 2015

Figure 68
CLINICAL SITUATION FOR QUESTIONS 68 AND 69
Figure 68 is the radiograph of a 17-year-old girl who is treated with closed
reduction and percutaneous screw fixation for her displaced femoral neck
fracture. Capsulotomy of the hip is not performed. After surgery, the patient
is instructed to maintain touch-down weight-bearing status for 3 months.
Question 68 of 101
Which technical error most likely could contribute to a poor outcome for this
patient?
1234-

Failure to perform a capsulotomy


Failure to use a fixed-angle device
Failure to achieve reduction of the fracture
Failure to protect postoperative weight bearing

PREFERRED RESPONSE: 3 fracture

Failure to achieve reduction of the

74

Musculoskeletal Trauma Scored and Recorded Self-Assessment Examination 2015

Question 69 of 101
The patient is at highest risk for which complication?
1234-

Postoperative infection
Heterotopic ossification
Malunion
Nonunion

PREFERRED RESPONSE: 4 -

Nonunion

DISCUSSION
The quality of femoral neck fracture reduction was the key outcome factor in
a number of studies. Capsulotomy is performed when achieving open
reduction of the femoral neck. In rare cases in which acceptable closed
reduction is achieved capsulotomy has been advocated, but poor anatomic
reduction is more likely to negatively influence the outcome. Although some
biomechanical data suggest that fixed-angle implants may be advantageous,
this has not been demonstrated in well-controlled clinical studies. If
reduction is not achieved, protecting postoperative weight bearing is not
likely to improve outcomes.
Failure to achieve anatomic reduction of the femoral neck frequently leads to
nonunion and varus collapse. Postoperative infection and/or heterotopic
ossification are not typically seen in closed reduction and percutaneous
stabilization of femoral neck fractures. Nonunion is more common than
malunion of displaced femoral neck fractures.

75

Musculoskeletal Trauma Scored and Recorded Self-Assessment Examination 2015

Musculoskeletal Trauma Scored and Recorded Self-Assessment


Examination 2015

Figure 70
Question 70 of 101
A 36-year-old man was injured in a motorcycle collision and sustained the
injury shown in Figure 70. He has a blood pressure (BP) of 70/40 mm Hg,
pulse of 148 beats per minute (bpm), and Glasgow Coma Scale score of 6
(scores lower than 8 indicate severe brain injury), and there is negligible
urine output. His airway is secure and intravenous (IV) access is obtained.
Two liters of warm crystalloid solution are given; repeated vital signs reveal
the same BP and a pulse of 142 bpm. What is the best next step?
1 - Administer IV fluids and then reassess vital signs before making further
decisions
2 - Pelvic binder and IV fluids
3 - Pelvic binder and immediate transfusion
4 - Pelvic binder, IV fluids, type and cross-match, and then transfuse
PREFERRED RESPONSE: 3 - Pelvic binder and immediate transfusion

DISCUSSION
This patient has an anteroposterior compression pelvic fracture associated
with shock. In patients with closed pelvic fractures and hypotension,
mortality rises to approximately 1 in 4 (10%-42%) and hemorrhage is the
76

Musculoskeletal Trauma Scored and Recorded Self-Assessment Examination 2015

major reversible contributing factor. Initial management of a major pelvic


disruption associated with hemorrhage requires hemorrhage control and
rapid fluid resuscitation. A pelvic binder should be placed to reduce pelvic
volume. The patient has signs and symptoms of class IV hemorrhage, which
include marked tachycardia exceeding 140, a significant decrease in BP, and
a very narrow pulse pressure. Urinary output is negligible, and mental status
is markedly depressed. The skin is cold and pale. The degree of
exsanguination with class IV hemorrhage is immediately life threatening,
and rapid transfusion and immediate surgical intervention are necessary.
Nonresponse to fluid administration indicates persistent blood loss. Blood
preparation should be emergency blood release. Type and cross-match of
blood can be used for additional resuscitation in transient responders.

77

Musculoskeletal Trauma Scored and Recorded Self-Assessment Examination 2015

Musculoskeletal Trauma Scored and Recorded Self-Assessment Examination 2015

Figure 71a

Figure 71c

Figure 71b

Figure 71d

Figure 73a
Figure 72a

Figure 73b

Figure 72b

Figure 74a
78

Figure 74b

Musculoskeletal Trauma Scored and Recorded Self-Assessment Examination 2015

RESPONSES FOR QUESTIONS 71 THROUGH 74


1 - Retrograde intramedullary (IM) nailing
2 - Open reduction and internal fixation (ORIF) with screws alone
3 - Locking condylar plate
4 - Circular external fixation
5 - Lateral and medial plates
Which treatment option listed is best for each patient described?
Question 71 of 101
A 54-year-old healthy man with the condition seen in Figures 71a through 71d
1 - Retrograde intramedullary (IM) nailing
2 - Open reduction and internal fixation (ORIF) with screws alone
3 - Locking condylar plate
4 - Circular external fixation
5 - Lateral and medial plates
PREFERRED RESPONSE: 3 - Locking condylar plate
Question 72 of 101
A 65-year-old polytrauma patient with the injury seen in Figures 72a and
72b
1 - Retrograde intramedullary (IM) nailing
2 - Open reduction and internal fixation (ORIF) with screws alone
3 - Locking condylar plate
4 - Circular external fixation
5 - Lateral and medial plates
PREFERRED RESPONSE: 1 - Retrograde intramedullary (IM) nailing

79

Musculoskeletal Trauma Scored and Recorded Self-Assessment Examination 2015

Question 73 of 101
32-year-old with the injury seen in Figures 73a and 73b
1 - Retrograde intramedullary (IM) nailing
2 - Open reduction and internal fixation (ORIF) with screws alone
3 - Locking condylar plate
4 - Circular external fixation
5 - Lateral and medial plates
PREFERRED RESPONSE: 3 - Locking condylar plate

Question 74 of 101
25-year-old with the injury seen in Figures 74a and 74b
1 - Retrograde intramedullary (IM) nailing
2 - Open reduction and internal fixation (ORIF) with screws alone
3 - Locking condylar plate
4 - Circular external fixation
5 - Lateral and medial plates
PREFERRED RESPONSE: 2 - Open reduction and internal fixation
(ORIF) with screws alone

DISCUSSION
Figures 71a through 71d reveal a severe intra-articular distal femur fracture
that is best treated with ORIF with a locking condylar plate. A retrograde IM
nail is not an ideal option for this application. Lateral and medial nonlocking
plates have gone by the wayside in favor of locked plating and fixed-angle
devices. External fixation will not allow for articular reconstruction and is
best reserved for temporary stabilization of these fractures. Screws alone
will not address this injury
Figures 72a and 72b reveal an extra-articular distal femur fracture that is
best treated with an IM nail, which would also allow for earlier weight
80

Musculoskeletal Trauma Scored and Recorded Self-Assessment Examination 2015

bearing. Screw fixation alone is inappropriate, and this does not necessitate
medial and lateral plate fixation. Although a locking condylar plate could be
used, blood loss in a polytrauma patient may be problematic. ?
Figures 73a and 73b show a comminuted supracondylar femur fracture with
complex intra-articular involvement. This would be treated using the same
application as seen in Figures 71a through 71d.
Figures 74a and 74b reveal a coronal plane fracture of the medial femoral
condyle, which can be treated with screws alone.

81

Musculoskeletal Trauma Scored and Recorded Self-Assessment Examination 2015

Musculoskeletal Trauma Scored and Recorded Self-Assessment


Examination 2015

Figure 75a

Figure 75b

Figure 75c

Figure 75d

Question 75 of 101
Figures 75a through 75d show the radiographs of an 85-year-old woman
who fell from a step and sustained a right proximal femur fracture. Six
months after surgery she has knee pain. What is the most likely cause of her
pain?
1 - Nail radius of curvature
2 - Osteoarthrosis
82

Musculoskeletal Trauma Scored and Recorded Self-Assessment Examination 2015

3 - Nonunion of fracture
4 - Improper starting point for nail
PREFERRED RESPONSE: 1 - Nail radius of curvature

DISCUSSION
Three cases of anterior distal femoral cortex penetration during
intramedullary nailing for subtrochanteric fractures were documented by
Ostrum and Levy in a 2005 study. Case 1 involved a Zimmer (Warsaw,
Indiana) M/DN antegrade femoral nail, Case 2 involved a Stryker (Mahwah,
New Jersey) long-stem Gamma nail, and Case 3 a DePuy Synthes (West
Chester, Pennsylvania) titanium femoral nail with spiral blade locking. The
anterior Zimmer nail penetration resulted in a displaced supracondylar
fracture, which subsequently required revision. The Gamma nail as well as
the DePuy Synthes nail were left impaled through the distal femoral cortex,
and the subtrochanteric fractures went on to union. The anteroposterior
radius of curvature for the Zimmer, long Gamma, and DePuy Synthes nails
is 257 cm, 300 cm, and 150 cm, respectively. It is estimated that the radius
of curvature of the femoral diaphyseal canal is 114 to 120 cm. It appears that
the difference in femoral anteroposterior bow between the bone and the
implant is a contributing factor to distal femoral anterior cortex penetration
in intramedullary nailing of subtrochanteric fractures. There is no evidence
of osteoarthrosis on the radiographs. Although nonunion is possible, based
on the radiographic findings it is more likely that this patient's pain is
attributable to the curvature of the nail. The lateral image of the hip reveals
an appropriate starting point for the device.

83

Musculoskeletal Trauma Scored and Recorded Self-Assessment Examination 2015

Musculoskeletal Trauma Scored and Recorded Self-Assessment


Examination 2015

Figure 76a

Figure 76b

Figure 78b
Figure 78a
RESPONSES FOR QUESTIONS 76 THROUGH 78
1 - Nonsurgical treatment with closed reduction and immobilization
2 - Early mobilization with physical therapy initiated within 2 weeks
3 - Open reduction and internal fixation (ORIF) with locked implants
4 - Use of supplemental bone graft or substitutes
5 - Arthroplasty
6 - Workup for osteoporosis and counseling
Which intervention listed is most appropriate to address each scenario
described?
84

Musculoskeletal Trauma Scored and Recorded Self-Assessment Examination 2015

Question 76 of 101
A 75-year-old man fell on his outstretched hand and sustained the fracture
seen in Figures 76a and 76b. What is the preferred initial treatment?
1 - Nonsurgical treatment with closed reduction and immobilization
2 - Early mobilization with physical therapy initiated within 2 weeks
3 - Open reduction and internal fixation (ORIF) with locked implants
4 - Use of supplemental bone graft or substitutes
5 - Arthroplasty
6 - Workup for osteoporosis and counseling
PREFERRED RESPONSE: 1 - Nonsurgical treatment with closed
reduction and immobilization

Question 77 of 101
In addition to the treatment plan, which intervention would also benefit this
patient?
1 - Nonsurgical treatment with closed reduction and immobilization
2 - Early mobilization with physical therapy initiated within 2 weeks
3 - Open reduction and internal fixation (ORIF) with locked implants
4 - Use of supplemental bone graft or substitutes
5 - Arthroplasty
6 - Workup for osteoporosis and counseling
PREFERRED RESPONSE: 6 - Workup for osteoporosis and counseling

85

Musculoskeletal Trauma Scored and Recorded Self-Assessment Examination 2015

Question 78 of 101
Figures 78a and 78b are the emergency department radiographs of an 83year-old woman who tripped and braced herself against a wall; this was
followed by shoulder pain. Which intervention would provide optimal
treatment for this patient?
1 - Nonsurgical treatment with closed reduction and immobilization
2 - Early mobilization with physical therapy initiated within 2 weeks
3 - Open reduction and internal fixation (ORIF) with locked implants
4 - Use of supplemental bone graft or substitutes
5 - Arthroplasty
6 - Workup for osteoporosis and counseling
PREFERRED RESPONSE: 2 - Early mobilization with physical therapy
initiated within 2 weeks

DISCUSSION
Low-energy fractures in elderly patients typically are treated with
nonsurgical care that involves early immobilization followed by early
rehabilitation/therapy, especially when proximal humerus and distal
humerus fractures are involved. Physical therapy should be initiated within
the first 2 weeks. If surgery is needed ORIF is preferred for most fractures,
but replacement may improve outcomes for unreconstructable fractures. The
use of hemiarthroplasty vs reverse shoulder replacement is currently being
debated.
For treatment of distal radius fractures in elderly patients, cast
immobilization for about 6 weeks will allow for optimal fracture healing.
This should be followed by aggressive therapy to improve range of motion
and function. Moderately displaced fractures in elderly patients will result in
satisfactory outcomes even though reduction may not be "anatomic."

86

Musculoskeletal Trauma Scored and Recorded Self-Assessment Examination 2015

Musculoskeletal Trauma Scored and Recorded Self-Assessment


Examination 2015

Figure 79a

Figure 79b

Figure 80a

Figure 79c

Figure 80b

87

Musculoskeletal Trauma Scored and Recorded Self-Assessment Examination 2015

Figure 81

Figure 82a

Figure 83a

Figure 82b

Figure 83b

RESPONSES FOR QUESTIONS 79 THROIUGH 83


1 - Percutaneous screw fixation
2 - Open reduction and internal fixation (ORIF) with a lateral plate
3 - ORIF with a posteromedial plate
4 - Dual plating
Which definitive surgical plan listed best addresses each injury pattern
described?

88

Musculoskeletal Trauma Scored and Recorded Self-Assessment Examination 2015

Question 79 of 101
40-year-old with the fracture seen on CT images in Figures 79a through 79c
1 - Percutaneous screw fixation
2 - Open reduction and internal fixation (ORIF) with a lateral plate
3 - ORIF with a posteromedial plate
4 - Dual plating
PREFERRED RESPONSE: 3 - ORIF with a posteromedial plate

Question 80 of 101
35-year-old with the injury pattern seen on the radiographs in Figures 80a
and 80b
1 - Percutaneous screw fixation
2 - Open reduction and internal fixation (ORIF) with a lateral plate
3 - ORIF with a posteromedial plate
4 - Dual plating
PREFERRED RESPONSE: 4 - Dual plating

Question 81 of 101
52-year-old with the injury seen on the CT image in Figure 81
1 - Percutaneous screw fixation
2 - Open reduction and internal fixation (ORIF) with a lateral plate
3 - ORIF with a posteromedial plate
4 - Dual plating
PREFERRED RESPONSE: 2 - Open reduction and internal fixation
(ORIF) with a lateral plate

89

Musculoskeletal Trauma Scored and Recorded Self-Assessment Examination 2015

Question 82 of 101
43-year-old with the injury pattern seen on the radiographs in Figures 82a
and 82b
1 - Percutaneous screw fixation
2 - Open reduction and internal fixation (ORIF) with a lateral plate
3 - ORIF with a posteromedial plate
4 - Dual plating
PREFERRED RESPONSE: 1 - Percutaneous screw fixation

Question 83 of 101
32-year-old with the injury pattern seen on the left lower extremity in CT
images in Figures 83a and 83b
1 - Percutaneous screw fixation
2 - Open reduction and internal fixation (ORIF) with a lateral plate
3 - ORIF with a posteromedial plate
4 - Dual plating
PREFERRED RESPONSE: 2 - Open reduction and internal fixation
(ORIF) with a lateral plate

DISCUSSION
In Figures 79a through 79c, CT images show a Schatzker IV medial tibial
plateau fracture. This fracture is best treated with a medial incision and
posteromedial plate, which will function as an antiglide or buttress plate.
Percutaneous screw fixation is insufficient for this injury. Lateral fixation is
not needed, and in many instances will not capture the medial fragment even
with locking fixation. ?
In Figures 80a and 80b, the patient has a fairly well-aligned bicondylar tibial
plateau fracture. The lateral joint is significantly depressed, necessitating
open reduction and elevation with stabilization. The posteromedial fragment
90

Musculoskeletal Trauma Scored and Recorded Self-Assessment Examination 2015

is often missed with single lateral locked plating and is best treated with a
posteromedial plate. Percutaneous fixation does not address the joint
depression.
In Figure 81, the patient has a classic split depression lateral tibial plateau
fracture (Schatzker II). The joint must be reduced from a lateral approach
and then supported and stabilized. Definitive ORIF with a lateral plate can
be performed when appropriate.
In Figures 82a and 82b, the radiographs show a Schatzker I nondisplaced
tibial plateau fracture that is amenable to percutaneous screw fixation if
surgical intervention is required. The fracture pattern can be treated
surgically, although, considering the comminution at the inferior aspect, late
displacement could occur. The other surgical approaches mentioned are not
required for this injury.
In Figures 83a and 83b, a lateral split depression tibial plateau fracture is
noted (similar to the fracture seen in Figure 81). The same logic applies.

91

Musculoskeletal Trauma Scored and Recorded Self-Assessment Examination 2015

Musculoskeletal Trauma Scored and Recorded Self-Assessment


Examination 2015
Question 84 of 101
A 19-year-old man was in a motorcycle accident. He sustained a grade IIIB
open tibia fracture with a wide zone of injury to the surrounding soft tissue
and a closed-head injury. The patient was treated emergently with irrigation,
debridement, and external fixation. What is the most accurate statement
regarding long-term functional and financial outcomes?
1 - Patients undergoing limb reconstruction are more satisfied.
2 - Long-term functional outcomes are superior in the amputation group.
3 - The percentage of patients who undergo amputation and return to work at
2 years is higher than the percentage of patients who undergo limb salvage
who return to work at 2 years.
4 - The cost of amputation is 3 times higher than the cost of limb
reconstruction.
PREFERRED RESPONSE: 4 - The cost of amputation is 3 times higher
than the cost of limb reconstruction.

DISCUSSION
Lower Extremity Assessment Project data suggest that long-term functional
outcomes and patient satisfaction at 7 years are equivalent between those
who undergo limb-salvage and primary amputations. Return to work is
essentially the same between the 2 groups. The projected lifetime healthcare
cost for patients treated with amputation is nearly 3 times higher than costs
for those who are treated with limb-salvage procedures.

92

Musculoskeletal Trauma Scored and Recorded Self-Assessment Examination 2015

Musculoskeletal Trauma Scored and Recorded Self-Assessment


Examination 2015

Figure 85a

Figure 85b

Figure 87b

Figure 87a

Figure 87c

CLINICAL SITUATION FOR QUESTIONS 85 THROUGH 88


Figures 85a and 85b are the plain radiographs of a 38-year-old man who fell
off the roof of a 2-story house and sustained an isolated injury to his right
knee. Examination reveals a swollen leg with a knee effusion. The skin is
intact, but there are some abrasions and an obvious deformity. His
neurovascular examination reveals active dorsiflexion and plantar flexion
with some pain and symmetric palpable pulses.
93

Musculoskeletal Trauma Scored and Recorded Self-Assessment Examination 2015

Question 85 of 101
How would you best classify this injury according to the Schatzker
classification?
1234-

Schatzker II
Schatzker IV
Schatzker V
Schatzker VI

PREFERRED RESPONSE: 4 -

Schatzker VI

Question 86 of 101
What is the best next step?
1 - Obtain CT images.
2 - Obtain ankle brachial indices.
3 - Obtain oblique views.
4 - Perform knee bridging external fixation.
PREFERRED RESPONSE: 4 - Perform knee bridging external fixation.

Question 87 of 101
CT images are shown in Figures 87a through 87c. In addition to fixation of
the tibial tubercle, what is the best treatment plan for this injury?
1234-

Medial and lateral incisions with dual plates


Midline incision with a lateral nonlocking plate
Midline incision with dual locking plates
Lateral incision with a lateral locking plate

PREFERRED RESPONSE: 1 plates

Medial and lateral incisions with dual

94

Musculoskeletal Trauma Scored and Recorded Self-Assessment Examination 2015

Question 88 of 101
A medial plate is best used to treat tibial plateau fractures when there is
1234-

comminution of the lateral side.


posteromedial fracture fragment.
metaphyseal comminution.
a lateral open fracture.

PREFERRED RESPONSE: 2 -

posteromedial fracture fragment.

DISCUSSION
The patient's plain radiographs demonstrate a bicondylar tibial plateau
fracture with complete separation of the diaphysis from the epiphysis,
making this a Schatzker VI injury. Clinically, the patient is neurovascularly
intact with symmetric palpable pulses, and ankle brachial indices are not
necessary.
There is significant lateral tibial plateau displacement with the lateral
femoral condyle down into the plateau defect. Considering the swelling,
abrasions, and severity of the injury, a bridging external fixator is warranted
followed by CT imaging.
The ligamentotaxis will provide better definition of the injury and joint
fragments and allow for soft-tissue rest and subsidence of the swelling for
eventual surgical intervention. Oblique views will not add as much
information as CT imaging, which will show the bicondylar nature of the
injury and the proximal tibia essentially split centrally with the tubercle as a
separate fragment. A midline incision with medial and lateral plating has
fallen out of favor secondary to wound-healing complications. Comminution
of the metaphysis or the lateral side is not an absolute indication for a medial
plate. Open lateral fractures can still be managed with a laterally based plate
depending on the soft-tissue injury.

95

Musculoskeletal Trauma Scored and Recorded Self-Assessment Examination 2015

Musculoskeletal Trauma Scored and Recorded Self-Assessment


Examination 2015

Figure 89a

Figure 89b

CLINICAL SITUATION FOR QUESTIONS 89 THROUGH 91


Figures 89a and 89b are the radiographs of an 80-year-old woman who fell
from a standing height.
Question 89 of 101
When treating this patient, it is important to be familiar with her
1 - history of other fractures such as hip or distal radius fractures.
2 - history of athletic participation.
3 - marital and social status.
4 - symptoms in the contralateral shoulder.
PREFERRED RESPONSE: 1 - history of other fractures such as hip or
distal radius fractures.
Question 90 of 101
When a fragility fracture is suspected, the workup should include
1234-

calcium and vitamin D levels.


a skeletal survey.
a bone scan.
urine analysis.

PREFERRED RESPONSE: 1 -

calcium and vitamin D levels.


96

Musculoskeletal Trauma Scored and Recorded Self-Assessment Examination 2015

Question 91 of 101
Use of long-term (at least 5 years) bisphosphonate medications for
osteoporosis is associated with
1 - increased risk for atypical femoral fractures.
2 - increased risk for femoral neck fractures.
3 - decreased risk for proximal humerus and wrist fractures.
4 - decreased risk for mandible osteonecrosis.
PREFERRED RESPONSE: 1 - increased risk for atypical femoral
fractures.
DISCUSSION
Fragility or osteoporotic fractures are common in postmenopausal women
and older men and include fractures of the proximal humerus and distal
radius and hip and vertebral compression fractures. The most reliable
predictor of a fragility fracture is a past fragility fracture; consequently, it is
important to elicit history. Assessment of vitamin D and calcium levels is
important when treating these patients because many of them have low
levels.
Use of bisphosphonates for treatment of osteoporosis results in atypical
femur fractures and a decrease in risk for fragility fractures (including hip
fractures). There also are reports of osteonecrosis of the mandible with
prolonged use of these drugs. Current medical literature suggests stopping
these drugs after 3 to 5 years to allow bone remodeling.

97

Musculoskeletal Trauma Scored and Recorded Self-Assessment Examination 2015

Musculoskeletal Trauma Scored and Recorded Self-Assessment


Examination 2015
CLINICAL SITUATION FOR QUESTIONS 92 THROUGH 95
A 23-year-old man is involved in a motorcycle collision and sustains a
displaced transcervical femoral neck fracture with an associated open
comminuted femoral shaft fracture.
Question 92 of 101
The patient is hypotensive and requires resuscitation, laparotomy, and
splenectomy. He improves immediately but continued resuscitation is
needed and his lactate level is 6.2 (reference range, 5.0-15 mg/dL). What is
the most appropriate next step?
1 - Debridement of the open fracture wound and percutaneous fixation of the
femoral neck fracture.
2 - Debridement of the open fracture wound and external fixation of the
femur.
3 - Debridement of the open femur fracture, open reduction and internal
fixation (ORIF) of the femoral neck via a Smith-Peterson approach, and
external fixation of the femoral shaft.
4 - Closed cephalomedullary nailing of the femur.
PREFERRED RESPONSE: 2 - Debridement of the open fracture wound
and external fixation of the femur
Question 93 of 101
A vertically oriented femoral neck fracture is unique because this pattern
1 - is more commonly found in elderly patients.
2 - favors bony union because the relatively longer fracture provides an
increased surface area for healing.
3 - is biomechanically advantageous because more compression occurs at
the fracture site.
4 - is seen more commonly with associated femoral shaft fractures.
PREFERRED RESPONSE: 4 - is seen more commonly with associated
femoral shaft fractures.
98

Musculoskeletal Trauma Scored and Recorded Self-Assessment Examination 2015

Question 94 of 101
The patient undergoes fixation of the associated femoral neck and shaft
fractures. The most appropriate fixation construct is a
1 - piriformis entry reconstruction nail to fix both fractures.
2 - long proximal femur locking plate to fix both fractures.
3 - cannulated screw fixation of the femoral neck with retrograde nail
fixation of the femoral shaft.
4 - antegrade nail fixation of the femoral shaft with cannulated screw
fixation placed around an antegrade nail.
PREFERRED RESPONSE: 3 - cannulated screw fixation of the femoral
neck with retrograde nail fixation of the femoral shaft.

Question 95 of 101
An anatomic reduction is obtained at the femoral neck. The most likely
reason for development of avascular necrosis (AVN) in this scenario would
be
1 - an ORIF delay exceeding 24 hours because of hemodynamic ?instability.
2 - an associated femur fracture.
3 - patient age and mechanism of injury.
4 - treatment with a closed reduction.
PREFERRED RESPONSE: 3 - patient age and mechanism of injury.

DISCUSSION
A damage-control approach is indicated for this patient. Debridement of the
open fracture wound and rapid stabilization without an extensive surgical
approach are indicated. Rapid percutaneous fixation of the femoral neck
would compromise long-term outcomes for this displaced fracture because
obtaining a quality reduction and fixation construct is critical for the longterm outcome. This patient likely would not tolerate cephalomedullary
nailing or open approaches very well at this time.
99

Musculoskeletal Trauma Scored and Recorded Self-Assessment Examination 2015

A vertically oriented (Pauwels 3) femoral neck fracture is more common in


younger patients who sustain high-energy injuries. Because of the
mechanism of injury, many of these patients have associated injuries. This is
a biomechanically challenging fracture because the fracture is subject to
shear forces rather than compression, making it inherently unstable. This
type of fracture often necessitates different fixation strategies to counter
shearing forces, such as use of a transversely oriented (Pauwels) screw to
compress the fracture or a fixed-angle device.
The femoral neck fracture should be prioritized in this scenario. This does
not necessarily mean that the femoral neck should be repaired first, but the
strategy should emphasize optimal fixation of the femoral neck. It has been
demonstrated that this is less successful when using a single implant to
repair both fractures. It is possible to place femoral neck fixation around an
antegrade femoral nail; however, it is much more likely that optimal fixation
will be achieved with shaft fixation that does not obstruct placement of
fixation for the femoral neck.
AVN is more common among physiologically young patients after femoral
neck fractures. The higher energy of injury is a likely contributor. Closed
reduction has not been shown to increase the risk for AVN when an
anatomic reduction is obtained. A surgical delay of 24 hours does not cause
AVN. Patients with associated femoral shaft fractures are not at increased
risk for AVN; in fact, some studies have shown a relatively lower rate of
AVN when a femoral neck fracture is associated with a femoral shaft
fracture.

100

Musculoskeletal Trauma Scored and Recorded Self-Assessment Examination 2015

Musculoskeletal Trauma Scored and Recorded Self-Assessment


Examination 2015
RESPONSES FOR QUESTIONS 96 THROUGH 99
1 - Warfarin (Coumadin)
2 - Dabigatran (Pradaxa)
3 - Rivaroxaban (Xarelto)
4 - Apixaban (Eliquis)
Match the appropriate oral anticoagulant listed with the description.
Question 96 of 101
This medication is a vitamin K antagonist and can be reversed.
1 - Warfarin (Coumadin)
2 - Dabigatran (Pradaxa)
3 - Rivaroxaban (Xarelto)
4 - Apixaban (Eliquis)
PREFERRED RESPONSE: 1 - Warfarin (Coumadin)

Question 97 of 101
This medication, a direct thrombin inhibitor, may be excreted slowly in
patients with renal insufficiency.
1 - Warfarin (Coumadin)
2 - Dabigatran (Pradaxa)
3 - Rivaroxaban (Xarelto)
4 - Apixaban (Eliquis)
PREFERRED RESPONSE: 2 - Dabigatran (Pradaxa)

101

Musculoskeletal Trauma Scored and Recorded Self-Assessment Examination 2015

Question 98 of 101
This medication, a factor Xa inhibitor, offers the advantage of once-daily
dosing.
1 - Warfarin (Coumadin)
2 - Dabigatran (Pradaxa)
3 - Rivaroxaban (Xarelto)
4 - Apixaban (Eliquis)
PREFERRED RESPONSE: 3 - Rivaroxaban (Xarelto)

Question 99 of 101
This medication, a factor Xa inhibitor, currently is not approved for venous
thromboembolism (VTE) prophylaxis.
1 - Warfarin (Coumadin)
2 - Dabigatran (Pradaxa)
3 - Rivaroxaban (Xarelto)
4 - Apixaban (Eliquis)
PREFERRED RESPONSE: 4 - Apixaban (Eliquis)

DISCUSSION
Warfarin has a long clinical track record and is well known among most
physicians. It is a vitamin K antagonist that can be monitored with
prothrombin time (INR) testing and reversed with vitamin K and fresh
frozen plasma if needed. Newer oral anticoagulants are becoming more
common and offer the advantage of being rapidly active without a need for
monitoring. These oral anticoagulants are not reversible, which can
complicate the treatment of patients who present with bleeding or require
surgery. Dabigatran (Pradaxa) is a direct thrombin inhibitor that is approved
for stroke prevention in atrial fibrillation. It is not reversible, and a surgical
delay of 24 to 48 hours is recommended for all but emergent surgeries. A
longer delay is recommended with renal insufficiency. Rivaroxaban
102

Musculoskeletal Trauma Scored and Recorded Self-Assessment Examination 2015

(Xarelto) is an oral factor Xa inhibitor that is approved for atrial fibrillation


and the treatment of VTE and deep vein thrombosis prophylaxis. It offers the
advantage of daily dosing. It is not reversible and a surgical delay of 36 to
48 hours is recommended. Apixaban (Eliquis) is another factor Xa inhibitor
for which twice-daily dosing is required. It is currently approved for stroke
prevention in atrial fibrillation, and a surgical delay of 36 to 48 hours is
recommended.

103

Musculoskeletal Trauma Scored and Recorded Self-Assessment Examination 2015

Musculoskeletal Trauma Scored and Recorded Self-Assessment


Examination 2015

Figure 100
CLINICAL SITUATION FOR QUESTIONS 100 AND 101
A 55-year-old woman slipped on ice while getting out of her car and
sustained the injury shown in Figure 100.
Question 100 of 101
Which material is preferred to fill the metaphyseal defect during open
reduction and internal fixation?
1234-

Recombinant human bone morphogenetic protein (BMP)-2


BMP-7
Calcium phosphate
Polymethylmethacrylate

PREFERRED RESPONSE: 3 -

Calcium phosphate

104

Musculoskeletal Trauma Scored and Recorded Self-Assessment Examination 2015

Question 101 of 101


What is the primary mechanism of degradation of calcium phosphate
cement?
1234-

Macrophage-mediated degradation
Osteoclast-mediated degradation
Giant-cell-mediated degradation
Dissolution by interstitial fluid

PREFERRED RESPONSE: 2 -

Osteoclast-mediated degradation

DISCUSSION
To treat depressed tibial plateau fractures, surgeons must elevate the
depressed segments, which results in a metaphyseal bone void. This void can
be filled with multiple materials, including autograft and allograft bone. The
addition of BMP-2 or BMP-7 is not recommended to fill contained bone
defects. These materials are indicated for application between bone and soft
tissue and they do not provide the volume or strength to fill closed defects.
Calcium phosphates have been use to fill these voids, demonstrating
improved strength and resistance to subsidence when compared to autograft.
Although polymethylmethacrylate is commonly used to fill bone voids in
patients with cancer, it is not preferred in the setting of acute fracture
because of the exothermic reaction that may compromise local bone healing.
Although hydroxyapatite may be degraded by macrophage and giant-cellmediated processes, the calcium phosphate degrades mainly by an
osteoclast-mediated method. The calcium phosphate is not soluble in
interstitial fluid, so simple dissolution does not occur.

105

Vous aimerez peut-être aussi